Почему бьет током заземление: Заземление, принцип работы и для чего оно предназначено

Содержание

Заземление, принцип работы и для чего оно предназначено

Заземление, принцип работы и для чего оно предназначено

Заземление – это комплекс решений и устройств для защиты от поражения электрическим током и обеспечению работы защитной аппаратуры.

Электросеть — это основа современного мира. Почти вся современная бытовая техника работает от электричества, ведь это удобный источник энергии. Но есть и обратная сторона медали – высокая опасность поражения электрическим током. Без правильного подхода конструированию оборудования и проектированию электрических сетей электричество наделает больше беды чем пользы. Заземление – один из способов обеспечения безопасности.

Отечественные электросети имеют глухозаземленную нейтраль. Что это значит? Если рассмотреть этот вопрос упрощённо, то на электростанциях устанавливают трёхфазные генераторы. Их обмотки соединяют по схеме звезды. Точка соединения обмоток является нейтралью.

Если заземлить точку соединения звезды, как это показано на рисунке выше, то получится линия электропередач с глухозаземленной нейтралью.

Потенциал этой точки и нейтрального провода будет равен потенциалу земли.

Заземляющее устройство называют заземлителем. Обычно это три металлических штыря убитые в землю на одинаковом расстоянии друг от друга, находясь как бы в вершинах треугольника, при этом их соединяют между собой стальной полосой с помощью сварки. Длина штырей и их поперечное сечение рассчитывается под конкретные условия и требования к этому объекту.

Далее в здание заводят главную заземляющую шин и от неё прокладывают проводники к электрощитам и к электрооборудованию.

По виду заземление бывает защитным и рабочим. Как можно догадаться, защитное заземление выполняет функции защиты от поражения электрическим током, а рабочее – нужно для нормального функционирования электрооборудования.

Таким образом заземлением называют электрическое соединения корпуса электроприборов с заземлителем.

Почему бьёт током

Чтобы разобраться для чего нужно заземление, для начала разберёмся в каких случаях и почему нас бьет током. Главное, что нужно для протекания электрического тока – это разность потенциалов.

Это значит, что если вы стоите на полу и возьметесь за оголенный провод или другую токоведущую часть руками – то ток через ваше тело и пол стечёт в землю.

Внимание:

Переменный ток силой всего в 50 мА уже является опасным для человека.

А если вы обеими руками возьметесь за токоведущую часть и повисните на ней не касаясь земли, то скорее всего ничего не произойдёт, проверять это, конечно не стоит. Поэтому птиц не бьет током на проводах. Но вернёмся к разговору о заземлении. Как мы уже сказали, корпуса электроприборов заземляют. Для чего это нужно?

Проводка и другие узлы оборудования, такие как электродвигатели, ТЭНы и прочее в нормальном состоянии не имеют контактов фазы с корпусом прибора, металлорукавом или бронёй кабеля. Но в случае неполадок фаза может оказаться на корпусе. Это может произойти при повреждении изоляции обмоток двигателей и трансформаторов, пробоя диэлектрического слоя ТЭНов, повреждения изоляции соединительных проводов внутри прибора и кабельных линий.

В результате на корпусе окажется опасный потенциал, простым языком: корпус окажется «под фазой». Когда вы коснетесь его стоя босиком на плитке, бетонном и даже деревянном полу – вас ударит током. В худшем случае, это может привести к смерти.

Чаще всего такая ситуация возникает в результате частичного выхода из строя ТЭНов стиральных машин, водонагревательных баков, проточных нагревателей. А особенно ярко такое ощущается при одновременном касании стиральной машины и водопроводных и отопительных труб, или в случае с водонагревательным баком, когда вы принимаете душ или ванную вас, бьёт током.

Последняя проблема решается организацией системы уравнивания потенциалов (заземлением ванны и других металлических частей водопровода).

Если корпус поврежденного прибора заземлён – опасное напряжение стечет на землю и (или) сработает защитный прибор – устройство защитного отключения (УЗО) или автоматический выключатель дифференциального тока (дифавтомат).

Если корпус занулён – сработает обычный автомат, так как это будет коротким замыканием на корпус (ноль в данном случае).

Дифавтоматы и УЗО определяют утечку тока путём сравнения токов фазного и нулевого провода – если ток в фазе больше чем в нуле, значит ток втекает в землю, через заземляющий провод или через тело человека. Такие приборы срабатывают при дифференциальном токе (разнице токов) обычно в 10 мА и более.

Поэтому современный электрощит – это сложное устройство с большим набором коммутационных защитных приборов, а наличие заземления является обязательным во всех зданиях, построенных или отремонтированных после 2003 года. То есть в них должна быть проложена 3-проводная однофазная или 5-проводная трёхфазная электропроводка. Если вы хотите высказать своё мнение по вопросам заземления – пишите в комментариях об этом.

Ранее ЭлектроВести писали о сопротивление тела человека — от чего зависит и как может изменяться.

По материалам electrik.info.

Как работает заземление. Принцип работы заземления

Всем известно, что электричество – это неотъемлемый атрибут современного человека. Без использования электроэнергии невозможно включить чайник, чтобы попить чая или кофе, разогреть еду в микроволновке или посмотреть телевизор. Несмотря на незаменимость электричества, не стоит забывать и о его коварстве. Очень много неприятных случаев бывает при ударе током, бывают даже летальные ситуации.

Приветствую дорогие друзья и читатели сайта «Электрик в доме». Многие ощущали на себе неприятный удар током, когда случайно касались оголенного провода. Но в быту встречаются ситуации, когда человека может ударить током, даже если он дотрагивается к безобидному с виду бытовому прибору. Почему так происходит?

Как правило, такое случается, когда повреждается внутренняя изоляция и прибор не имеет заземления. В этом материале постараемся простым языком объяснить читателю, что такое заземление, как работает заземление и для чего оно необходимо.

От чего защищает заземление?

Основное предназначение заземления в электрической сети – это защита. Для работы электрических приборов в электропроводке предусмотрено два провода: фазный и нулевой.

Защита, которую обеспечивает заземление заключается в подключении третьего проводника, соединенного непосредственно с заземлителем который в свою очередь соединен с контуром заземления. Благодаря заземлению можно не беспокоиться о том, что возникшая по вине неисправности бытового прибора аварийная ситуация приведет к удару электрическим током кого либо из окружающих.

Друзья давайте разберемся, какие аварийные ситуации могут возникнуть и в чем заключается

принцип работы защитного заземления?

Опасность поломки электрического прибора заключается в том, что его корпус может оказаться под напряжением, тем самым сделав его опасным. Такое обстоятельство может возникнуть в том случае, если повреждается внутренняя изоляция. Например, когда провода прибора со временем ссыхаются или плавятся, и соприкасается с металлическим корпусом бытового прибора.

Визуально заметить такую аварийную поломку невозможно, однако достаточно дотронуться к электроплите или стиральной машинке, удар током пройдет незамедлительно.

У многих после таких ситуаций возникает вопрос:

как работает заземление, и может ли оно эффективно защитить. Сила такого удара может быть разной в зависимости от состояния человека и окружающих условий.

Что произойдет, если корпус не соединен с заземлением? Сама по себе такая поломка ничего собой не представляет. Стиральная машинка с пробитым корпусом как работала, так и будет работать. Она будет отлично выполнять свои функции, пока вы к ней не дотронетесь.

Все дело в том, что человек больше чем на 70% состоит из воды и является прекрасным проводником электричества. Когда вы стоите на полу или прикасаетесь к стене, то ваше тело может послужить проводником. При прикосновении к поврежденному корпусу ток начнет протекать через ваше тело в землю.

Конечно, можно избежать удара током, если одеть резиновые перчатки или обувь, но в доме так никто не ходит. Если у вас в доме нет заземления, и прибор бьется током, следует помнить, что даже невысокое напряжение может привести к плачевным обстоятельствам.

Величина в 50 мА уже является опасной для человека. Такое маленькое значение тока может привести к фибрилляции сердца и даже к смертельному случаю.

Для того чтобы не беспокоиться за свою жизнь и здоровье семьи важно, чтобы в доме было подключено заземление. В этом случае опасный потенциал, имеющийся на корпусе прибора, будет уходить в землю, защищая вас от удара. В этом заключается

принцип работы заземления. К тому же дополнительно заземлению рекомендуется устанавливать УЗО, которое отключит поврежденное оборудование при малейших утечках.

Принцип работы заземления

После того как приборы будут заземлены пробой внутренней изоляции нам не страшен. Если по каким-то причинам корпус прибора окажется под напряжением, возникнет короткое замыкание между фазой и заземлением. В результате чего сработает автоматический выключатель. Благодаря правильно установленному заземлению и срабатыванию автомата, человека не ударит током.

Однако здесь есть некоторые нюансы электротехники. Не всегда при пробое напряжения на корпус может выбить автомат и в таких случаях прекрасным помощником станет устройство защитного отключения.

Также хочется отметить тот факт, что при качественном монтаже заземляющего контура его сопротивление должно составлять 4 Ом, и если по каким-то причинам произойдет задержка в отключении автомата или он вовсе не отключится, потенциал на корпусе поврежденного прибора будет равен потенциалу заземлителя. В этом случае человека при касании током не ударит, так как разность потенциалов отсутствует.

Как работает заземление электрооборудования

Что касается жителей частного сектора, то в основном, на этих районах электричество на участки подводится воздушными линиями электропередач. Как правило, это двухпроводные линии, которые состоят из фазного и нулевого провода. В нашей стране линии электропередач оставляют желать лучшего, ведь на одном кабеле, идущем по основной линии, может быть много скруток.

Порывы ветра, падающие ветки и осадки могут в любой момент оборвать силовой кабель и если у вас в доме не установлена система защиты в виде заземления и устройства УЗО, то пострадать может не только владелец дома, но и вся его техника. Здесь установка заземления особенно актуальный вопрос.

Сегодня можно самостоятельно создать хорошую защиту для дома и создать заземление собственными руками, обеспечивая сохранность приборов и здоровья домочадцев.

Правильно изготовленная и установленная система защиты сможет уберечь электроприборы даже в момент обрыва линии идущей к дому. В настоящее время индивидуальная работа заземления дома в совокупности с УЗО считается популярными средствами защиты от удара током в собственном доме.

Работа заземления в частном секторе

В данном разделе разберем, как работает заземление на примере частного дома. Схема питания дома, изображенная на рисунке состоит из воздушной линии. Воздушная линия – двухпроводная, наиболее часто встречающаяся в частном секторе. Состоит из двух проводов фазного (на рисунке обозначен красным цветом) и нулевого (синего цвета). Нулевой провод является нулевым рабочим и защитным одновременно. То есть совмещенным проводником. В электротехнической литературе обозначается как PEN проводник.

Для того чтобы разделить этот проводник на два независимых рабочий и защитный, во вводном щите дома делается специальное ответвление на заземляющий контур. После этого с вводного щита выходит два нулевых проводника которые имеют разное назначение. Один из них рабочий ноль, который служит для работы приборов. Другой защитный ноль — заземляющий проводник, должен иметь желто-зеленую маркировку и обозначение PE.

В «Правилах Устройства Электроустановок» такая система заземления обозначается как TN-C-S. Внутренняя электропроводка дома должна быть трехпроводной, то есть фаза, ноль и заземление. Все розетки в доме должны быть соответственно с заземляющим контактом. В этом случае корпус потенциально опасного прибора будет подключен к защитному проводнику через заземляющий контакт розетки. В зону риска особенно входит так называемая мокрая техника это водонагреватели, насосы, посудомоечные и стиральные машинки.

Если в ходе эксплуатации фазный провод в результате пробоя изоляции соприкасается с корпусом прибора (для примера это корпус холодильника), то между фазным проводом (красным) и заземляющим (желто-зеленым) произойдет замыкание, в результате чего отключится силовой автомат.

Мнимая защита или неправильное заземление

Бывают ситуации, когда заземление может быть опасным. Это при условии НЕПРАВИЛЬНОГО ПОДКЛЮЧЕНИЯ. Друзья сейчас рассмотрим случай неправильного подключения заземления и сравним его со случаем рассмотренным выше.

На рисунке изображена схема неправильного заземления. Суть его заключается в подключении заземляющего проводника (провода заземления в электропроводке) к нулевому рабочему. Нулевой провод же заземлен на подстанции, почему же от него не заземлиться? К сожалению, встречаются специалисты в нашей отрасли, которые совершают такие ошибки.

В чем заключается опасность? В исправном состоянии техника будет работать без нареканий, все электрические приборы будут выполнять свою работу. Друзья давайте теперь рассмотрим другую ситуацию когда нулевой провод на линии был оборван в результате сильного ветра, при этом красный все еще остался целым.

При замыкании фазного провода на корпус в этом случае короткого замыкания не возникнет, так как заземляющий провод, который одновременно является и нулевым рабочим оборван по пути к дому, разности потенциалов между фазным и заземляющим проводом нет, и короткого замыкания не произойдет. Отсюда не сложно догадаться, что автоматический выключатель не отключится, так как ему просто не на что реагировать (нет тока короткого замыкания).

Из этого следует, что корпус холодильника, находясь под опасным напряжением, будет ждать свою жертву. Сила удара током в этой ситуации будет напрямую зависеть от того какая соприкосаемость человека с землей. Чем лучше контакт, тем сильнее ударит.

В некоторых случаях удар током через корпус прибора может быть фатальным, чтобы не случилось неприятностей нужно знать, как работает заземление в доме.

К примеру, вы прикасаетесь к пробиваемой электрической водогрейке и одновременно беретесь за водопроводную трубу. Также опасно браться за корпус прибора, который находится под напряжением при этом стоять босым на бетонных полах. Такой пол может служить проводником.

Как работает узо с заземлением

Чувствительность системы заземления, а соответственно и электробезопасность можно повысить установив в электрощите устройство защитного отключения (УЗО). Данный прибор реагирует на утечку тока и отключается при ее появлении тем самым обестачивая технику с поврежденной изоляцией. УЗО срабатывает даже в тех случаях если происходит малейшая утечка тока.

В реальности утечка тока может происходить как через заземленный корпус прибора, так и через тело человека (если заземления в доме отсутствует), что менее приятно. На рисунке показана ситуация когда ток проходит через тело человека.

К примеру, человек касается корпуса неисправного прибора, корпус которого не заземлен. В момент прикосновения через человека начинает протекать ток, и УЗО реагируя на него мгновенно отключится. Продолжительность удара током для человека в этом случае будет равна времени отключения УЗО. Обычно она равняется десятым долям секунды.

Незначительное и кратковременное воздействие тока в большинстве случаев приносить незначительный вред, человек получает болевые неприятные ощущения и испуг, который проходит уже через несколько минут.

Казалось бы идеальный вариант защиты, но не все так гладко. Даже такая система защиты имеет свои недостатки:

  • если прибор не имеет заземления, то, следовательно, УЗО не сможет зафиксировать утечку, а понять поломку можно будет только после пусть небольшого, но удара током;
  • по сути УЗО — это сложный электронный прибор, который не может сработать моментально, для отключения требуется время, следовательно, защита только с помощью УЗО может оказаться слишком медленной.
  • за счет высокой стоимости на УЗО домовладельцы, как правило, экономят и покупают устройства низкого качества либо устанавливают одно УЗО на весь дом, а в этом случае сложно гарантировать своевременное срабатывание.

Не стоит использовать устройства УЗО сомнительного качества и малоизвестных брендов. Ответственность за свою защиту, каждый человек несет самостоятельно, поэтому покупать нужно только оригинальный и сертифицированный товар. В настоящий момент рынок переполнен электрооборудованием различных производителей и нужно ответственно относиться, к такой покупке.

Друзья мы с вами рассмотрели принцип работы заземления, и что может произойти при неправильном способе заземления. Основное преимущество такой схемы подключения заключается в том, что у нее имеется свой индивидуальный контур заземления и в случае обрыва провода на линии электропередач он не сможет никак повлиять на работоспособность.

Важно! Не стоит думать, что если у дома есть заземление, то не нужно использовать УЗО. Даже при малейшей утечке прибор может зафиксировать проблему и отключить поврежденный участок сети, обеспечив безопасность и здоровье человека.

Электричество – это друг и враг человека, поэтому чтобы не произошло чего-то непредвиденного необходимо правильно делать электропроводку, и знать, как работает заземление в доме. Если нет знаний и опыта работы с электричеством, то такую работу лучше доверить профессионалам, которые все сделают, не только быстро, но и качественно с учетом всех норм и требований.

Понравилась статья — поделись с друзьями!

 

Утечка тока и заземления в частном доме и квартире

В нашей практике очень часто бывают случаи, когда люди обращаются с проблемой, которая, на профессиональном языке, называется утечкой тока. Это явление происходит, когда ток от фазного провода проходит к земле, по непредназначенному для этого пути.

Люди же, не знающие специфической терминологии, говорят, например, что их бьет током, когда они моются в ванной, или, когда дотрагиваются до работающей стиральной машины. Бывает даже такое, что бьет током от обычной батареи центрального отопления.

Все эти факторы, говорят о том, что в вашей сети электроснабжения или, может быть, в сети целого жилого многоквартирного дома есть проблемы с утечкой тока.

Причины появления тока утечки.

Вариантов почему это может происходить – множество.

Утечка тока в стиральной машине, посудомоечной машине может происходить из-за изношенности внутренних изоляций проводов или из-за намокания проводки, проходящей в стене или полу.

Утечка тока через батареи центрального отопления может происходить из-за ошибок электрика, обслуживающего ваш дом. В редких случаях (но такие тоже бывали в нашей практике), некоторые умельцы заземляли свои электроприборы на центральное отопление или, присоединяясь к нему, пытались воровать электричество.

Следует помнить, что такие действия не только противозаконны, но и могут угрожать здоровью людей. В России за прошлый год погибло несколько десятков человек из-за таких махинаторов.

Устранение тока утечки

Все эти действия должен выполнять квалифицированный электрик!

Первое, что нужно сделать для устранения явления утечки тока – выявить места пробоя изоляции или места, где происходит соприкосновение фазного провода с корпусом прибора.

Второе – обеспечить надежное заземление всех приборов и устройств, которые возможно заземлить.

Третье – установить ШДУП (шина дополнительного уравнивания потенциалов) в ванных комнатах. Это делается для выравнивания разности потенциалов, между, например, корпусом чугунной ванны и стальными трубами системы водоснабжения.

Четвертое – установить в квартирном или домовой электрощите защиту от токов утечки, которыми являются устройство защитного отключения (УЗО) и дифференциальные автоматы. Данные приборы не устранят причину возникновения тока утечки, но помогут избежать удара электрическим током.

Следует заметить, что выбор номинального тока и тока утечки УЗО или диф. автомата тоже нужно доверить специалисту!

Обращайтесь к нам! Специалисты нашей компании не только устранят причины тока утечки, но и бесплатно проверят состояние вашей электрической сети.

На корпусе вашего компьютера напряжение 110 Вольт / Хабр

— У меня ноутбук бьется током, чувствую легкое покалывание. Не знаешь в чем дело?

Когда я в десятый раз услышал спор о причинах этого явления в кругу программистов с макбуками, стало понятно, что пора писать статью. Иногда этот эффект проявляется как легкая вибрация при соприкосновении кожи и металлических частей ноутбука, иногда как покалывание.

Короткий ответ: корпус вашего компьютера находится под напряжением ~110V (половина от напряжения в сети), но из-за маленькой силы тока вас не ударяет слишком сильно.

Для инженеров-электриков это банальность: по тем же причинам в домах со старой проводкой может бить током стиральная машина, когда касаешься ванны, корпус стационарного компьютера и т.д. Эта тема многократно поднималась в интернете, но до сих пор большинство людей не знает о причинах этого явления. Ситуация осложняется тем, что конструкция блока питания в европейских макбуках не позволяет избавиться от этого явления!

Почему это происходит?


Обычно неприятные ощущения покалывания возникают, когда человек касается каких-то заземленных металлических поверхностей, например радиатора батареи под столом и одновременно держит руки на металлической части компьютера. В моем случае это была заземленная металлическая кромка столешницы. Если одновременно коснуться кромки столешницы и макбука, в руках появлялось ощутимое покалывание.

И это вполне нормальная ситуация. Дело в том, что в схеме блока питания компьютера есть фильтр помех, вход фильтра выполнен на двух конденсаторах, подсоединенных с одной стороны на каждый из проводов сети 220вольт, а с другой их общая точка присоединена к корпусу. В результате получается делитель напряжения 220 вольт пополам. Отсюда появляется 110 вольт на корпусе.


Упрощенная схема фильтра помех компьютерного блока питания

На картинке выше показана упрощенная схема фильтра помех в блоке питания. Как видно, оба конденсатора подключены к защитному заземлению (желтый провод E), который в свою очередь подключен к корпусу устройства. Если блок питания подключен в розетку без заземления, то на корпусе появляется половинное напряжение от напряжения в сети. При этом ток в этой цепи протекает небольшой, но его вполне достаточно чтобы вызывать неприятные ощущения или небольшое искрение, если касаться его другим устройством с правильным заземлением. Так можно наблюдать маленькие искры при попытке соединить два устройства кабелем в случаях, когда одно из них подключено в розетку с заземлением, а другое без.

Блоки питания Apple

Как мы уже выяснили, напряжение на корпусе появляется только в случае подключения приборов в розетку без заземления. Таких розеток много в домах со старой проводкой, где заземление в розетках попросту отсутствует.


Однако даже в зданиях с современной проводкой, где в розетках есть правильно подключенное заземление, макбуки почему-то продолжают биться током. Все дело в особенностях блоков питания Apple.


Контакт заземления на блоке питания от макбука. Этот контакт связан с корпусом ноутбука.

Все блоки питания макбуков имеют съемные вилки для разных стран. Можно возить с собой в путешествия только маленький переходник и менять его при необходимости. В комплекте с макбуком всегда находится короткая вилка, которая вставляется сразу в корпус и длинная вилка на проводе. Так вот в европейских, американских и китайских коротких вилках отсутствует контакт заземления. Он есть только в британской вилке.


Короткая европейская вилка Apple не имеет контакта заземления

UPD: британская короткая вилка тоже не имеет контакт заземления внутри, хотя штекер заземления есть. Пруф.

И только удлиненная вилка с кабелем имеет контакт заземления. Это можно проверить, заглянув в место крепления вилки-насадки к блоку питания, внутри должны быть контакты, зажимающие шайбу заземления. Если их нет, ноутбук гарантированно будет биться током. Такое часто встречается на китайских поддельных блоках питания, даже на удлиненной розетке с кабелем.


Контакт заземления внутри съемной вилки

Заключение

Несмотря на банальность этой проблемы, мне постоянно приходится слышать новые теории ее происхождения, даже среди IT-шников. Если погуглить, находятся десятки тем, где люди жалуются на макбук под напряжением. Эта же проблема справедлива и для айфонов, подключенных к зарядному устройству.

1. How to properly ground a MacBook Pro
2. Electric shock coming from the edges of my macbook
3. MacBook Pro at 220 volts, could feel current through aluminum case

Если вас беспокоит эта проблема вот пара советов:

  • Проверьте, что в вашей розетке присутствует заземление. Иногда удлинители могут не иметь контакта заземления, хотя в розетке в стене он есть.
  • Используйте оригинальный блок питания макбука. Многие поддельные блоки питания не имеют контакта заземления
  • Используйте удлиненную вилку с кабелем. Проверьте, что контакт земли на вилке, которую вы вставляете в розетку, соединен с корпусом ноутбука или телефона.

UPDATE: Видео демонстрация

В комментариях и в личку мне написало несколько десятков человек, уверяя что показанное на картинке выше невозможно и больше напряжения выдаваемого блоком питания макбука (20V) разность потенциалов быть не может. Выкладываю видео с демонстрацией ТОГО САМОГО макбука с тем же самым мультиметром, лежащего на том же кухонном столе, на котором была сделана фотография.

Your browser does not support HTML5 video.

Стиральная машина бьет током: причины, что делать

Бытовые приборы, выполняющие значительную часть работы по дому, стали для нас настолько привычным явлением, что мы порою забываем о том, что пылесосы, печки, стиральные и посудомоечные машины – это сложная техника, которая нуждается в соответствующем обращении. Вспоминаем о правилах безопасности мы часто только тогда, когда один из наших помощников по хозяйству начинает вести себя странным образом, например, ударять пользователя электрическим током.

В этой статье мы хотим рассказать вам о том, почему это происходит с таким видом бытовой техники, как стиральные машины: из-за чего можно получить разряд электрического тока и какие меры следует принять, чтобы этого больше не происходило?

Причины

Причин, по которым стиралка начинает накапливать электричество и делиться им с домочадцами, может быть несколько. Иногда они бывают безобидными и не грозящими серьезными последствиями, но чаще эти причины следует незамедлительно устранить, чтобы избежать дальнейших неприятностей. Рассмотрим наиболее распространенные случаи.

Отсутствие заземления

При организации электрической проводки в квартире очень важно не забыть о заземляющем элементе. Заземление нужно именно для того, чтобы установленные дома приборы не накапливали электричество. Решить эту проблему можно единственным способом – подключить заземление хотя бы в том помещении, в котором находится стиральная машина.

Неисправность электропроводки внутри устройства

Эта причина относится к внутренним, так как связана непосредственно с неполадками в работе стиральной машинки. Диагностику прибора можно провести самостоятельно: сняв панели, под которыми скрывается блок управления, и проверив целостность проводов. Если повреждение не очень серьезное, его можно устранить при помощи изоленты или термотрубки.

Поломка одной из частей стиральной машины

Удары электрическим током может также вызвать неисправность важных элементов механизма, таких как двигатель или нагреватель. Чтобы выявить проблемное место, нужно разобрать прибор и провести тщательный визуальный осмотр всех его частей. В некоторых случаях неисправную деталь удается отремонтировать, но чаще ее приходится заменять.

Что делать?

Приведем пару наиболее действенных рекомендаций, которые дают электрики и мастера по ремонту бытовой техники тем, кто столкнулся с проблемой генерации тока стиральной машиной.

  • Установите на электрическую проводку в квартире устройство защитного отключения: при перепадах напряжения оно поможет предотвратить короткое замыкание, заблокировав подачу электричества.
  • Позаботьтесь о том, чтобы снабдить электропроводку системой уравнивания потенциалов. Она нужна для того, чтобы обнулять разницу между потенциалами проводников электрического тока. В квартире такими проводниками являются электроприборы и элементы коммуникаций. Уравновесив потенциалы, мы существенно понижаем риск поражения током.

Как правильно сделать заземление?

Чаще всего стиральная машина начинает бить электрическим током из-за того, что на электропроводке отсутствует заземляющий элемент или он подключен неверно.

Вот краткая инструкция по подключению заземления к стиральной машинке:

  • При монтаже электрической розетки, к которой будет подключена стиральная машина, присоедините к ней трехжильный провод, проведенный от электрощитка.
  • При этом должны быть установлены отдельные шины для земли и для ноля.
  • Подключите жилы: фазу – к автомату, ноль – к первой шине, землю – ко второй.

Если вы не обладаете достаточными знаниями и навыками в этой области, лучше обратиться за помощью к профессиональному электрику: только в этом случае вы будете уверенным в собственной безопасности и безопасности своей семьи.

Что делать нельзя?

  • Довольно часто обыватели решают проблему отсутствия заземления, просто подстилая под стиральную машину резиновый коврик. Следует знать, что это – лишь временная мера, которая не гарантируют полной безопасности.
  • Еще один довольно распространенный способ борьбы с накоплением электричества – это отключение сетевого фильтра прибора. Так вы сможете только снизить силу разряда, но от поражения током не убережетесь.
  • Использовать в качестве заземления своеобразный проводник, проведенный от стиралки к какому-либо металлическому элементу, например, батарее отопления. Делать этого ни в коем случае не стоит – так опасность воздействия электрического тока на человека лишь повышается.

Советы

  • Стиральная машинка должна находиться как можно дальше от сантехнических приборов, чтобы на нее не попадали капли воды. Розетка, к которой она подключена, обязательно должна быть влагозащищенной.
  • Специалисты советуют не оставлять стиралку постоянно подключенной к сети. Отключайте ее сразу же после завершения стирки и включайте только после того, как загружено белье и засыпан стиральный порошок.

Заземление стиральной машины своими руками | Советы мастера

Иногда даже новая стиралка бьет током. Почему возникает такая ситуация и что делать? Стоит ли производить заземление стиральной машины своими руками? Будем разбираться вместе.

Новые приборы и старые электросети

Сегодня производители выпускают бытовую технику, рассчитанную на работу в трехпроводных сетях системы TN-S. Этот способ подразумевает монтаж электропроводки с применением отдельного проводника заземления PE на всем протяжении сети.

В многоэтажных домах советской постройки электрические сети подключены по системе TN-C, в которой используется только два провода – фаза и ноль. Защита от короткого замыкания осуществляется подключением нулевого провода к заземляющему контуру на подстанции.

Устаревшая система не позволяет корректно подключить современную бытовую технику по правилам безопасной эксплуатации. Новые приборы оснащены фильтром снижения высокочастотных помех, в котором на корпус техники подключен средний потенциал встроенных конденсаторов. В трехпроводной системе TN-S корпус заземлен через контакт в розетке, поэтому потенциал по заземляющему проводнику PE уходит в землю.

При подключении к двухпроводной сети заземляющего контакта в розетке нет, напряжение 110 вольт остается на корпусе. Такой потенциал не может нанести вреда здоровью, рука ощущает только легкие пощипывания. Если же совпадет несколько неблагоприятных факторов, то даже такое напряжение может привести к серьезной травме. Например, если одной рукой дотронуться до корпуса стиральной машины, а другой – взяться за трубу или смеситель.

Следует либо заземлить стиральную машину по схеме, предусмотренной производителем, либо отключить вывод средней точки конденсаторного фильтра. Не стоит откладывать это дело «на потом». Проблему надо решать сразу.

Защита от поражения током в трехпроводной сети

В современных стиральных машинах корпус всегда подсоединен к контуру земли. Если фаза подключилась на корпус, ток уходит из питающей сети. В этом случае срабатывает устройство защитного отключения (УЗО). Прибор сравнивает величину токов идущих к потребителю и возвращающихся по нулевому проводу. При коротких замыканиях возникает дисбаланс, УЗО отсекает потенциалы фазы и ноля.

Защита от поражения током в двухпроводной сети

В системе TN-C устройство защитного отключения работает менее эффективно. Так как заземляющий контур отсутствует, при прикосновении к корпусу, находящемуся под напряжением, ток пройдет через тело человека в землю. УЗО фиксирует уменьшение потенциала на нулевом проводе и отключает подачу, сократив время протекания тока через тело человека. По этой причине установка УЗО оправдана даже в устаревших сетях.

Способы заземления стиральной машины

В новых жилых комплексах квартиры подключены по схеме TN-S. Достаточно правильно подсоединить розетку к сети, заземление будет работать. Та же ситуация со схемой TN-C-S. Отличие только в том, что заземляющая шина соединяется с проводом ноля не на подстанции, а в общедомовом электрощите, впрочем, для владельца жилья это не имеет никакого значения.

Если квартира подключена к сети по двухпроводной схеме TN-C, не стоит пытаться сделать заземление своими руками. Заземляя бытовую технику к трубам водоснабжения или отопления, вы рискуете. Например: соседи снизу могут поменять металлические трубы на полипропиленовые и цепь будет разорвана.

Важно: Нельзя ставить в розетке перемычку, соединяя заземление с рабочим нулевым проводом в подключении по схеме TN-C. В случае случайной перемены проводов фазы и ноля в общедомовом электрощите вы получите напряжение 220 вольт на корпусе своей стиральной машины!

Ноль бьет током. 220 проседает. — Электрика

У меня был случай:
До инцидента напряжение держалось около 225 В, под нагрузкой практически не просаживалось. Меня все устраивало.
Начали менять на моей улице столбы и провода.
—————
Тут краткое лирическое отступление:
Бригада работников за три дня умудрилась:
1. порвать чужую машину — она им мешала, её потянули, а под днищем был пенек — оторвали задний мост.
2. неправильно поставили столб, а когда его вытаскивали, то уронили его на газовую магистраль. Сломали арку газовой трубы над дорогой. Оставили весь поселок без газа на один день. Развалили забор моего соседа.
3. одного работника люлькой вышки прижали к столбу — слава Богу без последствий.
4. после подачи напряжения на замененную линию, выяснилось, что в несколько домов напряжение не поступает. Жители на своих машинах догнали эту бригаду и заставили все переделывать.
Лирическое отступление окончено. 🙂
————————-
Теперь что получается после замены проводов. Напряжение в сети около 220 В. Включаю чайник — напряжение падает до 200 В. Свет притухает. На «ноле» напряжение относительно моей личной «земли» около 3. .5 В.
Мне эта ситуация не понятна — до замены линии все было нормально, что могло случиться после замены проводов с алюминиевых на СИП… Тем более, что заменили всего метров 150 проводов…
В такой ситуации дома даже болгаркой не поработаешь…
Еду в местный РЭС, обрисовываю им ситуацию. В тот же вечер сюда приезжает бригада и начинает искать незаконные подключения. Ничего не находят. Проблема остается.
Через два снова еду в РЭС, снова обрисовываю им ситуацию. Днем приезжает бригада, лазит по столбам, ничего не находит. Их бригадир приходит и говорит, что все нормально, оснований для паники нет, так и должно быть. Проблема снова осталась.
Через два дня снова еду в РЭС, иду к их гл.инженеру, ему обрисовываю ситуацию. Он мне суёт под нос кучу бумажек, где написаны акты приемки-сдачи этой линии после замены, рапорта их бригад, где написано, что все нормально.
И мне он заявляет, что напряжение 200 вольт под нагрузкой входит в их допуски (кажется -20% и +10%).
На что я ему отвечаю, что они мне не напряжение они поставляют, а электроэнергию, а в этой энергии еще есть один параметр — это ток. А вот тока сейчас как раз-то и нет.
Сую ему свою корочку с работы, что у меня 4 гр.допуска и работаю инж.электроником. И обещаю дойти до области, есть не вернут все как было.
В тот же день приезжает еще одна бригада, уже другая и начинает интенсивно с вышкой лазить по столбам. Через час все исправляют.
Спрашиваю у ребят что было. Оказывается, бригада лабухов не проколола «орех»соединения «ноля» с моей улицы на магистральную линию.
А «ноль» у меня был потому, что на столбах есть повторное заземление. Оно маленькую нагрузку может выдержать, а вот мощную уже не тянет.
Так что проблема с «нолем», тут надо рыться.

Предотвращение удара с помощью надлежащих методов заземления — охрана труда и безопасность

Предотвращение поражения электрическим током с помощью надлежащих методов заземления

Восемьдесят процентов всех проблем с качеством электроэнергии обнаруживаются в системе распределения электроэнергии и заземления.

  • Чад Рейнольдс
  • 1 ноября 2003 г.

AN по оценкам 58 человек каждую неделю теряют жизнь в результате поражения электрическим током.В электрической системе система заземления является основной защитой от поражения электрическим током. Он обеспечивает заземление с низким сопротивлением для защиты от электрических повреждений. Использование надлежащих методов заземления, проверка и поддержание хорошего электрического заземления, а также установка защитных устройств — лучшие способы защитить людей и оборудование от поражения электрическим током.

Поддержание качественного заземления начинается с правильного подключения цепи. Национальный электрический кодекс (NEC) требует, чтобы удаление любого устройства не могло прервать заземление.Производители розеток отреагировали, поставив розетки только с одним заземляющим контактом. Это запретило бы электрикам подключать устройство последовательно с цепью заземления.



= «центр»>

Распространенным методом обеспечения целостности заземления является использование гибкого кабеля. Чтобы сделать косичку, возьмите оба провода заземления и соедините их 6-дюймовым проводом того же цвета, который был зачищен с обоих концов.Крепко возьмите все три и свяжите их вместе проволочным соединителем. Обязательно используйте разъем, размер которого соответствует размеру и количеству проводов.

Доступны специальные соединители, облегчающие эту работу. В одном из них через отверстие в верхней части разъема вставляется неизолированный медный провод. Затем все провода связывают, скручивая разъем до упора. Готовые косички становятся все более популярными из-за экономии времени. Например, в некоторых разъемах теперь совмещен скручивающийся провод с предварительно обжатым жгутом.Сверхгибкий 6-дюймовый провод обеспечивает беспроблемное размещение в распределительной коробке, а заземляющие кабели оснащены предварительно обжатым вилочным соединением для быстрой и простой установки устройства.


Эта статья была впервые опубликована в ноябрьском номере журнала «Охрана труда и безопасность» за 2003 год.

Заземление и соединение | Электробезопасность прежде всего

Почему нужно проверять заземление и соединение?

Если вы вносите дополнительные изменения в вашу электрическую установку, ваш электрик должен проверить (а также другие вещи), что имеющиеся у вас устройства заземления и соединения соответствуют требуемым стандартам.

Это связано с тем, что безопасность любой новой работы, которую вы выполняете (даже небольшой), будет зависеть от схемы заземления и соединения.

Что такое заземление?

Если в вашей электрической установке возникнет неисправность, вы можете получить удар электрическим током, если дотронетесь до металлической детали, находящейся под напряжением. Это потому, что электричество может использовать ваше тело как путь от токоведущей части к земной.

Заземление используется для защиты от поражения электрическим током. Это достигается путем обеспечения пути (защитного проводника) для тока короткого замыкания, протекающего на землю. Это также приводит к тому, что защитное устройство (автоматический выключатель или предохранитель) отключает электрический ток в цепи, в которой возникла неисправность.

Например, если плита неисправна, ток короткого замыкания течет на землю через защитные (заземляющие) проводники. Защитное устройство (предохранитель или автоматический выключатель) в потребительском блоке отключает электропитание плиты. Теперь плита защищена от поражения электрическим током любого, кто к ней прикоснется.

Что такое склеивание?

Склеивание используется для снижения риска поражения электрическим током любого, кто может прикоснуться к двум отдельным металлическим частям при неисправности где-то в электроснабжении электроустановки.Соединяя соединительные проводники между отдельными частями, он снижает возможное напряжение.

Обычно используются следующие типы склеивания: основное склеивание и дополнительное склеивание.

Дополнительные советы

Электрик даст вам совет, если ваше заземление или соединение необходимо улучшить по соображениям безопасности.

Мы настоятельно рекомендуем вам использовать электрика, зарегистрированного в утвержденной правительством схеме, для выполнения любых необходимых вам электромонтажных работ.

Для получения подробной информации о том, как найти зарегистрированного электрика, щелкните здесь.

Определения

Склеивание — Способ снижения риска поражения электрическим током.

Проводники — Провода, по которым проходит электричество.

Consumer Unit — Блок предохранителей, который используется для управления и подачи электричества в доме. Обычно он содержит главный выключатель, предохранители или автоматические выключатели и одно или несколько устройств защитного отключения (УЗО).

Ток — Текущее электричество.

Земля — ​​ Соединение с землей.

Заземление — Способ предотвращения поражения электрическим током.

Электромонтаж — стационарная электропроводка.

Live — Активный (есть электричество).

Основное соединение — Зеленые и желтые проводники, которые соединяют металлические трубы (газ, вода или масло) внутри здания с главной клеммой заземления электроустановки.Основные соединительные соединения также могут быть выполнены за пределами здания, например, если снаружи установлен полузакрытый газовый счетчик, и невозможно установить соединение с трубопроводом газовой установки в помещении.

Главный зажим заземления — Где заземляющий и соединительный проводники соединены вместе.

Устройства защитного отключения (УЗО) — Чувствительное переключающее устройство, отключающее цепь при обнаружении замыкания на землю.

Дополнительное соединение — Зеленые и желтые проводники, которые соединяют доступные металлические части электрического оборудования (например, полотенцесушитель) с доступными металлическими частями предметов электрического оборудования и / или доступными металлическими частями предметов, которые не являются электрическими (например, трубы). Эти соединения выполнены для предотвращения опасного напряжения между двумя доступными металлическими частями в случае неисправности. Вам может потребоваться дополнительное соединение для комнат, в которых есть ванна или душ, за исключением случаев, когда все цепи в комнате защищены УЗО, а основное соединение соответствует требуемым стандартам.

Напряжение — Сила электричества.

В чем разница между PE и FG?

Правильное заземление необходимо для электрических устройств по разным причинам, но зачем мы это делаем?

Моим первым неудачным опытом работы с электричеством было поражение электрическим током от розетки переменного тока.Я помню, как мое тело вибрировало около секунды. Излишне говорить, что я держался подальше от электричества, пока мне не пришлось подключать продукты, чтобы смоделировать реальные сценарии работы с клиентами в полевых условиях. Именно тогда я узнал, насколько на самом деле важно заземление.

Почему земля?

  • Предотвратить повреждение или травмы
  • Защита от электрической перегрузки
  • Стабилизировать уровни напряжения

Правильное заземление может предотвратить поражение электрическим током людей, работающих с электричеством. Электричество всегда проходит самый простой путь от напряжения до земли.

Пример стиральной машины ниже иллюстрирует концепцию пути прохождения тока в приборе, который не заземлен, а не заземлен.

Когда устройство не заземлено, ток утечки, генерируемый внутри устройства, становится потенциалом, который просто ищет путь к земле. Как только человек прикоснется к прибору и у него появится свободный путь к земле, он станет заземляющим проводом, и ток пройдет через человеческое тело, а затем на землю.Не знаю, можете ли вы сказать, но у нее не счастливое лицо.

Когда прибор заземлен, ток утечки теперь имеет менее устойчивый путь к земле, чем человеческое тело, поэтому ток утечки пропускает человеческое тело и проходит через заземляющий провод в вилке переменного тока, который имеет свой собственный путь к земле. Теперь у нее счастливое лицо.

Зачем нужно заземлять двигатели?

Ну, во-первых, заземление требуется практически для всех электродвигателей.Национальный электрический кодекс (NEC), раздел 430-L, определяет условия заземления двигателя.

Электроэнергия течет через обмотки двигателя, которые обычно изолированы от других частей двигателя. Потенциально опасная ситуация возникает при выходе из строя изоляции. В этот момент корпус двигателя может стать проводником при том же напряжении, подаваемом на двигатель. Любое прикосновение к корпусу двигателя и заземленной поверхности может стать причиной травмы или чего-то еще хуже. После заземления двигателя избыточное напряжение будет безопасно заземлено.

Поражение электрическим током или, что еще хуже, поражение электрическим током может произойти, если клемма PE двигателя не заземлена. Сила тока от 0,1 до 0,2 ампер потенциально может убить человека.

Почему на этом знаке всегда написано
«высокое напряжение» вместо «высокий ток»?

Давайте рассмотрим роли трех обычных подозреваемых по закону Ома, V, I и R, в поражении электрическим током.

Напряжение — это потенциальная энергия в виде электрического заряда, ток — это выходной сигнал в виде потока электрического заряда, который определяется в амперах, а сопротивление препятствует прохождению тока.

На самом деле ток — самый опасный из трех. Причина, по которой на табличке всегда написано «высокое напряжение», заключается в том, что без высокого напряжения не было бы достаточно тока, чтобы быть опасным.

Угроза переменного тока широко варьируется в зависимости от его частоты, тогда как постоянный ток просто становится более опасным по мере увеличения уровней напряжения и тока. Вот таблица OSHA, в которой описан потенциальный ущерб.

Что означают «PE» и «FG»?

PE — Защитное заземление

В Великобритании это называют «заземлением».В США мы называем это «заземлением». Они означают одинаковый электрический потенциал 0 В. Назначение полиэтилена — защита от поражения электрическим током и возгорания из-за тока утечки.

В то время как заземление двигателя раньше выполнялось одним из четырех болтов или винтов, теперь предлагаются специальные винтовые клеммы для упрощения реализации.

FG — Заземление рамы

Это также известно как «земля шасси». Назначение FG — защита от электрических помех, которые могут искажать сигналы и вызывать сбои в работе.

Примечание. В этом посте не обсуждается сигнальное заземление, которое является третьим типом заземления, которое обычно путают с защитным заземлением и заземлением корпуса. Для получения информации о сигнальном заземлении, пожалуйста, обратитесь к этой статье Основные правила: заземление, шасси и сигнальное заземление от Analog IC Tips.

Примеры клемм PE

Клемма PE может быть винтовой клеммой на двигателе или винтовой клеммой на приводе.И двигатель, и привод необходимо заземлить.

Примеры: клеммы PE

На приведенном ниже примере установки двигателя и драйвера и схемы подключения показано, где заземление PE необходимо в конфигурации системы шагового двигателя.

Для мер по предотвращению электрических помех, включая заземление FG, мы предоставляем следующую информацию в наших руководствах.

СОВЕТ: используйте более толстый и короткий заземляющий провод
При подключении заземляющего провода к заземлению используйте более толстый и короткий провод.Это снижает сопротивление провода, поэтому току легче протекать.

Для получения инструкций по заземлению вашего конкретного продукта Oriental Motor обратитесь к руководствам по эксплуатации или обратитесь к нашим полезным инженерам службы технической поддержки. Самый простой способ найти руководство по эксплуатации продукта — выполнить поиск по номеру детали. Нужна помощь? Свяжитесь с нами по телефону, электронной почте или в чате.

Подпишитесь, чтобы получать уведомления о новых сообщениях.

Выключатели и заземляющие провода

Термин «земля» относится к заземлению, которое действует как резервуар заряда. Заземляющий провод обеспечивает проводящий путь к земле, который не зависит от нормального пути прохождения тока в электрическом приборе. На практике в бытовых электрических цепях он подключается к электрической нейтрали на сервисной панели, чтобы гарантировать достаточно низкое сопротивление для отключения автоматического выключателя в случае электрического сбоя (см. Рисунок ниже).Прикрепленный к корпусу устройства, он удерживает напряжение корпуса при потенциале земли (обычно принимаемом за ноль напряжения). Это защищает от поражения электрическим током. Заземляющий провод и предохранитель или прерыватель являются стандартными устройствами безопасности, используемыми в стандартных электрических цепях.

Нужен ли заземляющий провод? Устройство будет нормально работать без заземляющего провода, поскольку он не является частью токопроводящей дорожки, по которой к устройству подается электричество.Фактически, если заземляющий провод сломан или удален, вы, как правило, не заметите разницы. Но если на корпус попадет высокое напряжение, может возникнуть опасность поражения электрическим током. При отсутствии заземляющего провода условия опасности поражения электрическим током часто не приводят к срабатыванию выключателя, если в цепи нет прерывателя замыкания на землю. Частично роль заземляющего провода состоит в том, чтобы заставить выключатель сработать, обеспечивая путь к земле, если «горячий» провод соприкасается с металлическим корпусом устройства.

В случае электрической неисправности, которая приводит к опасному высокому напряжению в корпусе устройства, вы хотите, чтобы автоматический выключатель немедленно отключился, чтобы устранить опасность. Если корпус заземлен, в заземляющем проводе прибора должен протекать большой ток, который отключит прерыватель. Это не так просто, как кажется — привязки заземляющего провода к заземляющему электроду, вбитому в землю, обычно недостаточно для срабатывания прерывателя, что меня удивило. U.S. Статья 250 Национального электротехнического кодекса требует, чтобы провода заземления были привязаны к электрической нейтрали на сервисной панели. Таким образом, при межфазном замыкании ток короткого замыкания протекает через провод заземления устройства к сервисной панели, где он присоединяется к нейтральному тракту, протекая через главную нейтраль обратно к центральному отводу сервисного трансформатора. Затем он становится частью общего потока, приводимый в действие служебным трансформатором в качестве электрического «насоса», который производит достаточно высокий ток короткого замыкания для отключения выключателя.В электротехнической промышленности этот процесс привязки заземляющего провода к нейтрали трансформатора называется «соединением», и суть в том, что для обеспечения электробезопасности вы должны быть одновременно заземлены и соединены.

Это лишь верхушка айсберга, важная для правильного заземления и соединения электрических систем. См. Сайт Майка Холта для получения дополнительной информации.

Индекс

Практические концепции схем


Майк Холт

Почему электрические цепи необходимо заземлять?

Электрические устройства «заземляются», когда они подключаются к заземляющим устройствам из соображений безопасности.Заземление обеспечивает безопасный «путь наименьшего сопротивления» для следования паразитному напряжению. Системы заземления направляют паразитное напряжение в землю, где оно безопасно разряжается, а не накапливается в опасных местах.

Без заземления скачки напряжения или повреждение оборудования могут сделать электрические цепи опасными или разрушительными. Они могут повредить подключенные электроприборы, поразить окружающих людей или даже вызвать пожар. Заземление — важный элемент безопасности для электрической системы любой конструкции.Фактически, электрические нормы требуют этого для всех новых построек. Вот что вы должны знать об электрическом заземлении и зачем оно вам нужно:

Как работает электрическое заземление?

Электрические цепи обычно содержат три провода: «горячий» провод, «нейтральный» провод и заземляющий провод. Горячий провод содержит активное напряжение, питающее электрические приборы. Активное напряжение заряжено отрицательно. Отрицательно заряженное электричество естественным образом стремится разрядить свою отрицательную энергию, чтобы вернуться в состояние нейтрального заземления.Для этого он проходит через нейтральный провод и возвращается к положительному заряду главной сервисной панели. Когда цепи замыкаются, отрицательные заряды проходят через горячий провод и возвращаются на землю через нейтральный провод. Если все исправно, заземляющий провод никогда не вступит в игру.

К сожалению, цепи могут ломаться или работать неправильно, как и все остальное. При повреждении или обрыве проводов в цепи электричество может выходить из системы и попадать в другие материалы.В зависимости от того, где протекает ток, он может шокировать вас, что-то повредить или вызвать пожар. Заземляющие провода предотвращают это. При поиске нейтрали напряжение всегда следует по пути наименьшего сопротивления, даже когда оно выходит из замкнутой системы. Провода заземления обеспечивают этот путь наименьшего сопротивления. Они подключаются к шине заземления в земле под вашей цепью. Когда в системе возникает паразитное напряжение, заземляющий провод «ловит» его и передает на землю, где он не может повредить вам.

Почему важно электрическое заземление?

Заземление электрических цепей — очень важная процедура безопасности. Заземление помогает защитить вас и ваш дом от опасностей повреждения цепей или электрических перегрузок. Когда случаются скачки напряжения, избыточное электричество, введенное в систему, может выскочить из проводки. Без электрического заземления это паразитное напряжение может вызвать возгорание, повредить приборы или шокировать окружающих.

Правильное заземление защитит электрическую систему вашего дома даже в случае сильного скачка напряжения или удара молнии.Заземление предотвращает возникновение электрической дуги на других проводящих материалах, таких как вода и металл, где оно может повредить вам. Заземляющие провода также предотвращают перегрузку напряжением и повреждение ваших приборов, что помогает им прослужить дольше и лучше работать. В целом, заземление — одна из важнейших мер безопасности, защищающих современные домашние электрические системы.

Как узнать, заземлены ли мои цепи?

Может быть трудно определить, правильно ли заземлен ваш дом, без тестера цепей или профессионального осмотра.Однако есть пара вещей, которые помогут вам составить представление. Прежде всего: у ваших розеток два или три контакта? Нижний третий контакт розетки подключается к заземляющему проводу. Если в вашем доме есть двухконтактные розетки, они представляют собой незаземленные электрические цепи. Если у вас есть двусторонние розетки, обновите их до GFCI ASAP .

Даже если каждая из ваших розеток имеет три контакта, у вас все равно может не быть эффективного заземления. Иногда дома, которые когда-то были заземлены, теперь имеют неэффективное заземление из-за повреждений или ошибок в электропроводке.Дома, построенные в 50-х и 60-х годах, часто не имеют заземления или неэффективного заземления, даже если в них есть трехконтактные розетки. Если вы хотите точно знать, заземлен ли ваш дом, приобретите домашний тестер цепей или запланируйте осмотр.

Что делать, если мои цепи не заземлены?

Лучше всего вызвать электрика для немедленной установки заземления. Специалисты Early Bird могут перемонтировать весь ваш дом, чтобы включить безопасное и современное заземление в каждую из ваших цепей.Если вы не хотите переделывать свой дом для заземления, вам следует как минимум заменить двухконтактные розетки на GFCI.

GFCI, или «прерыватели цепи замыкания на землю», могут обеспечить вам уровень защиты даже без заземления. Эти розетки автоматически прерывают подачу электричества при возникновении опасности поражения электрическим током, возгорания или повреждения. У вас должны быть розетки GFCI на кухне и в ванных комнатах вашего дома, независимо от того, заземлили вы свой дом или нет! Розетки GFCI не защитят вашу технику так же эффективно, как заземление, но они помогут защитить вас.

Если вы хотите установить заземление, заменить двухконтактные розетки на GFCI или у вас есть другой вопрос по электрике, свяжитесь с Early Bird Electric в любое время. Наши лицензированные и опытные специалисты помогут вам решить любые проблемы с электричеством. Мы хотим помочь сделать ваш дом безопасным.

ЭЛЕКТРИЧЕСКАЯ БЕЗОПАСНОСТЬ — прикладное промышленное электричество

Важность электробезопасности

С помощью этого урока я надеюсь избежать распространенной ошибки, обнаруживаемой в учебниках по электронике, состоящей в игнорировании или недостаточном освещении темы электробезопасности.Я предполагаю, что тот, кто читает эту книгу, хотя бы мимолетно заинтересован в реальной работе с электричеством, и поэтому тема безопасности имеет первостепенное значение.

Еще одно преимущество включения подробного урока по электробезопасности — это практический контекст, который он устанавливает для основных понятий напряжения, тока, сопротивления и схемы. Чем более актуальной будет техническая тема, тем больше вероятность того, что студент обратит внимание и поймет. А что может быть важнее приложения для личной безопасности? Кроме того, поскольку электрическая энергия является повседневным явлением в современной жизни, почти каждый может ознакомиться с иллюстрациями, приведенными на таком уроке.Вы когда-нибудь задумывались, почему птиц не шокируют, когда они отдыхают на линиях электропередач? Читайте и узнайте!

Физиологические эффекты электричества

Большинство из нас испытали ту или иную форму электрического «шока», когда электричество заставляет наше тело испытывать боль или травму. Если нам повезет, степень этого переживания ограничится покалыванием или приступами боли от накопления статического электричества, разряженного через наши тела. Когда мы работаем с электрическими цепями, способными передавать большую мощность нагрузкам, поражение электрическим током становится гораздо более серьезной проблемой, а боль — наименее значимым результатом поражения электрическим током.

Поскольку электрический ток проходит через материал, любое противодействие току (сопротивлению) приводит к рассеиванию энергии, обычно в виде тепла. Это самый простой и понятный эффект воздействия электричества на живую ткань: ток заставляет ее нагреваться. Если количество выделяемого тепла достаточно, ткань может обжечься. Эффект носит физиологический характер, такой же, как и повреждение, вызванное открытым пламенем или другим высокотемпературным источником тепла, за исключением того, что электричество обладает способностью сжигать ткани под кожей жертвы, даже обжигая внутренние органы.

Как электрический ток влияет на нервную систему

Еще одно воздействие электрического тока на тело, возможно, наиболее опасное, касается нервной системы. Под «нервной системой» я имею в виду сеть особых клеток в организме, называемых нервными клетками или нейронами, которые обрабатывают и проводят множество сигналов, ответственных за регуляцию многих функций организма. Мозг, спинной мозг и сенсорные / двигательные органы в теле функционируют вместе, позволяя ему чувствовать, двигаться, реагировать, думать и запоминать.

Нервные клетки взаимодействуют друг с другом, действуя как «преобразователи», создавая электрические сигналы (очень малые напряжения и токи) в ответ на ввод определенных химических соединений, называемых нейротрансмиттерами , и высвобождая эти нейротрансмиттеры при стимуляции электрическими сигналами. Если электрический ток достаточной силы проходит через живое существо (человека или другое), его эффектом будет подавление крошечных электрических импульсов, обычно генерируемых нейронами, перегрузка нервной системы и предотвращение способности рефлекторных и волевых сигналов действовать. задействовать мышцы.Мышцы, вызванные внешним (шоковым) током, непроизвольно сокращаются, и жертва ничего не может с этим поделать.

Эта проблема особенно опасна, если пострадавший касается руками проводника под напряжением. Мышцы предплечья, отвечающие за сгибание пальцев, как правило, лучше развиты, чем мышцы, отвечающие за разгибание пальцев, и поэтому, если оба набора мышц будут пытаться сокращаться из-за электрического тока, проводимого через руку человека, «сгибающие» мышцы выиграют, сжимая пальцы в кулак.Если проводник, подающий ток к пострадавшему, обращен к ладони его или ее руки, это сжимающее действие заставит руку крепко ухватиться за провод, тем самым ухудшая ситуацию, обеспечивая отличный контакт с проводом. Пострадавший совершенно не сможет отпустить проволоку.

С медицинской точки зрения это состояние непроизвольного сокращения мышц называется столбняком . Электрики, знакомые с этим эффектом поражения электрическим током, часто называют обездвиженную жертву поражения электрическим током «зависшей в цепи».Вызванный током столбняк можно прервать, только отключив ток через пострадавшего.

Даже когда ток прекращается, жертва не может восстановить произвольный контроль над своими мышцами в течение некоторого времени, поскольку химический состав нейротрансмиттера находится в беспорядке. Этот принцип был применен в устройствах «электрошокера», таких как электрошокеры, которые основаны на принципе мгновенного поражения жертвы высоковольтным импульсом, передаваемым между двумя электродами. Правильно нанесенный электрошокер временно (на несколько минут) обездвиживает жертву.

Однако электрический ток может воздействовать не только на скелетные мышцы жертвы электрошока. Мышца диафрагмы, контролирующая легкие, и сердце, которое само по себе является мышцей, также могут быть «заморожены» в состоянии столбняка электрическим током. Даже токи, слишком слабые для того, чтобы вызвать столбняк, часто способны перебивать сигналы нервных клеток настолько, что сердце не может биться должным образом, вызывая состояние, известное как фибрилляция . Фибриллирующее сердце скорее трепещет, чем бьется, и не может перекачивать кровь к жизненно важным органам тела.В любом случае смерть от удушья и / или остановки сердца обязательно наступит из-за достаточно сильного электрического тока, проходящего через тело. По иронии судьбы, медицинский персонал использует сильный разряд электрического тока, приложенный к груди жертвы, чтобы «подтолкнуть» фибриллирующее сердце к нормальному ритму биений.

Эта последняя деталь подводит нас к другой опасности поражения электрическим током, свойственной коммунальным энергосистемам. Хотя наше первоначальное исследование электрических цепей будет сосредоточено почти исключительно на постоянном токе (постоянный ток или электричество, которое движется в непрерывном направлении в цепи), современные энергетические системы используют переменный ток или переменный ток.Технические причины такого предпочтения переменного тока перед постоянным током в энергосистемах не имеют отношения к этому обсуждению, но особые опасности каждого вида электроэнергии очень важны для темы безопасности.

Воздействие переменного тока на организм во многом зависит от частоты. Низкочастотный (от 50 до 60 Гц) переменный ток используется в домашних хозяйствах США (60 Гц) и Европы (50 Гц); он может быть опаснее высокочастотного переменного тока и в 3-5 раз опаснее постоянного тока того же напряжения и силы тока. Низкочастотный переменный ток вызывает продолжительное сокращение мышц (тетанию), которое может прижать руку к источнику тока, продлевая воздействие.Постоянный ток, скорее всего, вызовет одиночное судорожное сокращение, которое часто заставляет жертву отойти от источника тока.

Переменный характер

AC имеет большую тенденцию приводить нейроны кардиостимулятора в состояние фибрилляции, тогда как DC имеет тенденцию просто вызывать остановку сердца. Как только ток разряда прекращается, у «замороженного» сердца больше шансов восстановить нормальный ритм сердечных сокращений, чем у фибриллирующего сердца. Вот почему «дефибриллирующее» оборудование, используемое врачами скорой помощи, работает: электрический разряд, подаваемый дефибриллятором, — это постоянный ток, который останавливает фибрилляцию и дает сердцу шанс восстановиться.

В любом случае электрические токи, достаточно высокие, чтобы вызвать непроизвольное мышечное действие, опасны, и их следует избегать любой ценой. В следующем разделе мы рассмотрим, как такие токи обычно входят в тело и выходят из него, и рассмотрим меры предосторожности против таких случаев.

  • Электрический ток может вызвать глубокие и серьезные ожоги тела из-за рассеяния мощности через электрическое сопротивление тела.
  • Столбняк — это состояние, при котором мышцы непроизвольно сокращаются из-за прохождения внешнего электрического тока через тело.Когда непроизвольное сокращение мышц, управляющих пальцами, приводит к тому, что жертва не может отпустить проводник под напряжением, жертва считается «замороженной в цепи».
  • Диафрагма (легкие) и сердечные мышцы одинаково подвержены действию электрического тока. Даже токи, слишком слабые, чтобы вызвать столбняк, могут быть достаточно сильными, чтобы мешать работе нейронов кардиостимулятора, заставляя сердце трепетать, а не сильно биться.
  • Постоянный ток (DC) с большей вероятностью вызовет столбняк в мышцах, чем переменный ток (AC), поэтому постоянный ток с большей вероятностью «заморозит» жертву в случае шока.Однако переменный ток с большей вероятностью вызовет фибрилляцию сердца жертвы, что является более опасным состоянием для жертвы после прекращения действия электрического тока.

Электричество требует полного пути (цепи) для непрерывного потока. Вот почему удар, полученный от статического электричества, представляет собой только мгновенный толчок: течение тока обязательно кратковременно, когда статические заряды уравниваются между двумя объектами. Подобные самоограниченные шоки редко бывают опасными.

Без двух точек контакта на теле для входа и выхода тока, соответственно, опасность поражения электрическим током отсутствует. Вот почему птицы могут спокойно отдыхать на высоковольтных линиях электропередачи, не подвергаясь электрошоку: они контактируют с цепью только в одной точке.

Рисунок 1.1

Для того, чтобы ток протекал по проводнику, должно присутствовать напряжение, которое его мотивирует. Напряжение, как вы должны помнить, всегда составляет относительно двух точек . Не существует такого понятия, как напряжение «на» или «в» одной точке цепи, и поэтому птица, контактирующая с одной точкой в ​​вышеуказанной цепи, не имеет напряжения, приложенного к ее телу, чтобы установить ток через нее.Да, даже если они опираются на две ноги , обе ступни касаются одного и того же провода, что делает их электрически общими . С точки зрения электричества, обе птичьи лапы соприкасаются с одной и той же точкой, поэтому между ними нет напряжения, которое могло бы стимулировать ток через тело птицы.

Это может привести к мысли, что невозможно получить поражение электрическим током, прикоснувшись только к одному проводу. Как птицы, если мы будем касаться только одного провода за раз, мы будем в безопасности, верно? К сожалению, это не так.В отличие от птиц, при контакте с «живым» проводом люди обычно стоят на земле. Часто одна сторона энергосистемы будет намеренно подключена к заземлению, и поэтому человек, касающийся одиночного провода, фактически устанавливает контакт между двумя точками в цепи (провод и заземление):

Рисунок 1.2

Значок земли представляет собой набор из трех горизонтальных полос уменьшающейся ширины, расположенных в нижнем левом углу показанной схемы, а также у ступни человека, подвергающегося электрошоку.В реальной жизни заземление энергосистемы представляет собой какой-то металлический проводник, закопанный глубоко в землю для обеспечения максимального контакта с землей. Этот проводник электрически подключен к соответствующей точке соединения в цепи толстым проводом. Заземление жертвы осуществляется через ноги, которые касаются земли.

В этот момент в уме ученика обычно возникает несколько вопросов:

  • Если наличие точки заземления в цепи обеспечивает легкую точку контакта для кого-то, чтобы получить электрошок, зачем вообще она в цепи? Разве схема без заземления не была бы безопаснее?
  • Человек, которого шокирует, вероятно, не ходит босиком.Если резина и ткань являются изоляционными материалами, то почему их обувь не защищает их, предотвращая образование цепи?
  • Насколько хорошим проводником может быть грязь ? Если вы можете быть поражены током, протекающим через землю, почему бы не использовать землю в качестве проводника в наших силовых цепях?

В ответ на первый вопрос, наличие намеренной точки «заземления» в электрической цепи должно гарантировать, что одна сторона безопасна для контакта.Обратите внимание, что если бы наша жертва на приведенной выше диаграмме коснулась нижней стороны резистора, ничего бы не произошло, даже если бы их ноги все еще касались земли:

Рис. 1.3

Поскольку нижняя сторона цепи надежно соединена с землей через точку заземления в нижнем левом углу цепи, нижний провод цепи электрически общий с заземлением. Поскольку между электрически общими точками не может быть напряжения, к человеку, контактирующему с нижним проводом, не будет подаваться напряжение, и они не получат удара током.По той же причине провод, соединяющий цепь с заземляющим стержнем / пластинами, обычно остается оголенным (без изоляции), так что любой металлический объект, о который он задевает, будет электрически общим с землей.

Заземление цепи гарантирует, что по крайней мере одна точка в цепи будет безопасна для прикосновения. Но как насчет того, чтобы оставить цепь полностью незаземленной? Разве это не сделало бы человека, касающегося только одного провода, таким же безопасным, как птица, сидящая только на одном? В идеале да. Практически нет.Посмотрите, что происходит без земли:

Рисунок 1.4

Несмотря на то, что ноги человека все еще соприкасаются с землей, любая точка в цепи должна быть безопасной для прикосновения. Поскольку не существует полного пути (цепи), образованного через тело человека от нижней стороны источника напряжения к верхней, нет возможности установить ток через человека. Однако все это может измениться из-за случайного заземления, например, если ветка дерева касается линии электропередачи и обеспечивает соединение с землей.Такое случайное соединение между проводником энергосистемы и землей (землей) называется замыканием на землю .

Рисунок 1.5

Замыкания на землю

Замыкания на землю могут быть вызваны многими причинами, в том числе скоплением грязи на изоляторах линий электропередач (создание пути грязной воды для тока от проводника к полюсу и к земле во время дождя), проникновением грунтовых вод в подземные проводники линии электропередач. , и птицы, приземляющиеся на линии электропередач, перемыкая линию к полюсу своими крыльями.Учитывая множество причин замыканий на землю, они, как правило, непредсказуемы. В случае с деревьями никто не может гарантировать , с какой проволокой могут касаться их ветви. Если бы дерево задело верхний провод в цепи, это сделало бы верхний провод безопасным для прикосновения, а нижний опасным — как раз противоположность предыдущему сценарию, когда дерево касается нижнего провода:

Рисунок 1.6

Когда ветвь дерева соприкасается с верхним проводом, этот провод становится заземленным проводом в цепи, электрически общим с заземлением.Следовательно, между этим проводом и землей нет напряжения, а есть полное (высокое) напряжение между нижним проводом и землей. Как упоминалось ранее, ветви деревьев являются лишь одним потенциальным источником замыканий на землю в энергосистеме. Рассмотрим незаземленную энергосистему без соприкосновения деревьев с деревьями, но на этот раз с двумя людьми, касающимися отдельных проводов:

Рис. 1.7

Когда каждый человек стоит на земле и контактирует с разными точками цепи, путь для электрического тока проходит через одного человека, через землю и через другого человека.Несмотря на то, что каждый человек думает, что он в безопасности, только коснувшись одной точки в цепи, их совместные действия создают смертельный сценарий. Фактически, один человек действует как замыкание на землю, что делает его небезопасным для другого человека. Именно поэтому незаземленные энергосистемы опасны: напряжение между любой точкой цепи и землей (землей) непредсказуемо, потому что замыкание на землю может возникнуть в любой точке цепи в любое время. Единственный персонаж, который гарантированно будет в безопасности в этих сценариях, — это птица, которая вообще не связана с землей! Надежно подключив обозначенную точку цепи к заземлению («заземлив» цепь), по крайней мере, безопасность может быть обеспечена в этой точке.Это большая гарантия безопасности, чем полное отсутствие заземления.

Отвечая на второй вопрос, обувь с резиновой подошвой или действительно обеспечивает некоторую электрическую изоляцию, чтобы помочь защитить кого-то от проведения электрического тока через ступни. Однако наиболее распространенные конструкции обуви не являются электрически «безопасными», поскольку их подошва слишком тонкая и не из подходящего материала. Кроме того, любая влага, грязь или токопроводящие соли из пота тела на поверхности подошвы или проникающие сквозь нее могут поставить под угрозу ту небольшую изоляционную ценность, которая должна была изначально иметь обувь.Есть обувь, специально предназначенная для опасных электромонтажных работ, а также толстые резиновые коврики, на которых можно стоять во время работы с цепями под напряжением, но эти специальные детали должны быть в абсолютно чистом и сухом состоянии, чтобы быть эффективными. Достаточно сказать, что обычной обуви недостаточно, чтобы гарантировать защиту от поражения электрическим током от электросети.

Исследования контактного сопротивления между частями человеческого тела и точками контакта (например, с землей) показывают широкий диапазон цифр (информацию об источнике этих данных см. В конце главы):

  • Контакт для рук или ног, с резиновой изоляцией: обычно 20 МОм.
  • Контакт ступни через кожаную подошву обуви (сухой): от 100 кОм до 500 кОм
  • Контакт ступни через кожаную подошву обуви (мокрый): от 5 кОм до 20 кОм

Как видите, резина не только является гораздо лучшим изоляционным материалом, чем кожа, но и присутствие воды в пористом веществе, таком как кожа , значительно снижает электрическое сопротивление.

Отвечая на третий вопрос, грязь — не очень хороший проводник (по крайней мере, когда она сухая!). У него слишком плохой проводник, чтобы поддерживать постоянный ток для питания нагрузки.Однако, как мы увидим в следующем разделе, требуется очень мало тока, чтобы ранить или убить человека, поэтому даже плохой проводимости грязи достаточно, чтобы обеспечить путь для смертельного тока при наличии достаточного напряжения, как обычно находится в энергосистемах.

Некоторые шлифованные поверхности лучше изолируют, чем другие. Например, асфальт на масляной основе имеет гораздо большее сопротивление, чем большинство видов грязи или камней. Бетон, с другой стороны, имеет довольно низкое сопротивление из-за внутреннего содержания воды и электролита (проводящего химического вещества).

  • Поражение электрическим током может произойти только при контакте между двумя точками цепи; когда на тело жертвы подается напряжение.
  • Цепи питания
  • обычно имеют обозначенную точку, которая «заземлена»: прочно соединена с металлическими стержнями или пластинами, закопанными в грязь, чтобы гарантировать, что одна сторона цепи всегда находится под потенциалом земли (нулевое напряжение между этой точкой и землей).
  • A замыкание на землю — это случайное соединение между проводником цепи и землей (землей).
  • Специальная изолированная обувь и коврики предназначены для защиты людей от ударов через заземление, но даже эти части снаряжения должны быть в чистом, сухом состоянии, чтобы быть эффективными. Обычная обувь недостаточно хороша, чтобы обеспечить защиту от ударов, изолируя ее владельца от земли.
  • Хотя грязь — плохой проводник, она может проводить достаточно тока, чтобы ранить или убить человека.

Распространенная фраза в отношении электробезопасности звучит примерно так: « Убивает не напряжение, а ток ! «Хотя в этом есть доля правды, об опасности поражения электрическим током нужно понимать больше, чем эта простая пословица.Если бы напряжение не представляло опасности, никто бы никогда не распечатал и не вывесил надписи: ОПАСНО — ВЫСОКОЕ НАПРЯЖЕНИЕ!

Принцип «убивает текущее» по существу верен. Это электрический ток, который сжигает ткани, замораживает мышцы и вызывает фибрилляцию сердца. Однако электрический ток не возникает сам по себе: должно быть доступное напряжение, чтобы побудить ток протекать через жертву. Тело человека также оказывает сопротивление току, что необходимо учитывать.

Взяв закон Ома для напряжения, тока и сопротивления и выразив его через ток для заданных напряжения и сопротивления, мы получим следующее уравнение:

[латекс] \ textbf {закон Ома} [/ латекс]

[латекс] Ток = \ frac {Напряжение} {Сопротивление} [/ латекс] [латекс] I = \ frac {E} {R} [/ латекс]

Величина тока, протекающего через тело, равна величине напряжения, приложенного между двумя точками этого тела, деленному на электрическое сопротивление, оказываемое телом между этими двумя точками.Очевидно, что чем больше напряжения доступно для протекания тока, тем легче он будет проходить через любое заданное сопротивление. Следовательно, существует опасность высокого напряжения, которое может генерировать ток, достаточный для получения травмы или смерти. И наоборот, если тело имеет более высокое сопротивление, меньший ток будет протекать при любом заданном напряжении. Насколько опасно напряжение, зависит от общего сопротивления цепи, препятствующего прохождению электрического тока.

Сопротивление тела не является фиксированной величиной.Это варьируется от человека к человеку и время от времени. Существует даже метод измерения содержания жира в организме, основанный на измерении электрического сопротивления между пальцами рук и ног. Различное процентное содержание жира в организме обеспечивает разное сопротивление: одна переменная, влияющая на электрическое сопротивление в организме человека. Чтобы методика работала точно, человек должен регулировать потребление жидкости за несколько часов до теста, что указывает на то, что гидратация тела является еще одним фактором, влияющим на электрическое сопротивление тела.

Сопротивление тела также варьируется в зависимости от того, как происходит контакт с кожей: от руки к руке, от руки к ноге, от ступни к ступне, от руки к локтю и т. Д. Пот, богатый солью и минералами. , являясь жидкостью, является отличным проводником электричества. То же самое и с кровью с таким же высоким содержанием проводящих химикатов. Таким образом, контакт с проводом потной рукой или открытой раной будет оказывать гораздо меньшее сопротивление току, чем контакт с чистой сухой кожей.

Измеряя электрическое сопротивление чувствительным измерителем, я измеряю примерно 1 миллион Ом (1 МОм) на руках, держась за металлические щупы измерителя между пальцами.Измеритель показывает меньшее сопротивление, когда я крепко сжимаю щупы, и большее сопротивление, когда я держу их свободно. Я сижу за компьютером и печатаю эти слова, мои руки чистые и сухие. Если бы я работал в жаркой, грязной промышленной среде, сопротивление между моими руками, вероятно, было бы намного меньше, представляя меньшее сопротивление смертельному току и большую опасность поражения электрическим током.

Насколько опасен электрический ток?

Ответ на этот вопрос также зависит от нескольких факторов.Химический состав тела человека оказывает значительное влияние на то, как электрический ток влияет на человека. Некоторые люди очень чувствительны к току, испытывая непроизвольное сокращение мышц из-за разряда статического электричества. Другие могут получить большие искры от разряда статического электричества и почти не почувствовать его, не говоря уже о мышечном спазме. Несмотря на эти различия, с помощью тестов были разработаны приблизительные руководящие принципы, которые показывают, что для проявления вредных эффектов требуется очень небольшой ток (опять же, информацию об источнике этих данных см. В конце главы).Все текущие значения даны в миллиамперах (миллиампер равен 1/1000 ампер):

ТЕЛО ВЛИЯНИЕ МУЖЧИНЫ / ЖЕНЩИНЫ ПРЯМОЙ ТОК (ПОСТОЯННЫЙ ТОК) 60 Гц 100 кГц
Легкое ощущение под рукой Мужчины 1,0 мА 0,4 мА 7 мА
Женщины 0,6 мА 0,3 мА 5 мА
Порог боли Мужчины 5.2 мА 1,1 мА 12 мА
Женщины 3,5 мА 0,7 мА 8 мА
Болезненные, но произвольный контроль мышц сохраняется Мужчины 62 мА 9 мА 55 мА
Женщины 41 мА 6 мА 37 мА
Болезненно, провода не отпускаются Мужчины 76 мА 16 мА 75 мА
Женщины 60 мА 15 мА 63 мА
Сильная боль, затрудненное дыхание Мужчины 90 мА 23 мА 94 мА
Женщины 60 мА 15 мА 63 мА
Возможная фибрилляция сердца через 3 секунды Мужчины и женщины 500 мА 100 мА

«Гц» означает блок Гц .Это мера того, насколько быстро изменяется переменный ток, иначе известный как частота . Таким образом, столбец цифр, обозначенный «60 Гц переменного тока», относится к току, который изменяется с частотой 60 циклов (1 цикл = период времени, когда ток течет в одном направлении, а затем в другом) в секунду. Последний столбец, обозначенный «10 кГц переменного тока», относится к переменному току, который совершает десять тысяч (10 000) возвратно-поступательных циклов каждую секунду.

Имейте в виду, что эти цифры являются приблизительными, поскольку люди с разным химическим составом тела могут реагировать по-разному.Было высказано предположение, что ток через грудную клетку всего 17 мА переменного тока достаточно, чтобы вызвать фибрилляцию у человека при определенных условиях. Большинство наших данных относительно индуцированной фибрилляции получены в результате испытаний на животных. Очевидно, что проводить тесты индуцированной фибрилляции желудочков на людях непрактично, поэтому имеющиеся данные отрывочны. О, и если вам интересно, я понятия не имею, почему женщины, как правило, более восприимчивы к электрическому току, чем мужчины! Предположим, я положил руки на клеммы источника переменного напряжения с частотой 60 Гц (60 циклов в секунду).Какое напряжение необходимо для этого состояния чистой, сухой кожи, чтобы получить ток в 20 миллиампер (достаточно, чтобы я не мог отпустить источник напряжения)? Мы можем использовать закон Ома, чтобы определить это:

[латекс] E = IR [/ латекс]

[латекс] E = (20 мА) (1 M \ Omega) [/ латекс]

[латекс] \ textbf {E = 20 000 вольт или 20 кВ} [/ латекс]

Имейте в виду, что это «лучший случай» (чистая, сухая кожа) с точки зрения электробезопасности, и что это значение напряжения представляет собой величину, необходимую для индукции столбняка.Чтобы вызвать болезненный шок, потребуется гораздо меньше! Кроме того, имейте в виду, что физиологические эффекты любой конкретной силы тока могут значительно отличаться от человека к человеку, и что эти расчеты являются приблизительными оценками , всего лишь .

Обрызгав пальцы водой, чтобы имитировать пот, я смог измерить сопротивление рук в руках всего 17 000 Ом (17 кОм). Имейте в виду, что это касается только одного пальца каждой руки, касающегося тонкой металлической проволоки. Пересчитав напряжение, необходимое для возникновения тока в 20 мА, мы получим эту цифру:

[латекс] E = IR [/ латекс]

[латекс] E = (20 мА) (17 кОмега) [/ латекс]

[латекс] \ textbf {E = 340 V} [/ латекс]

В этих реальных условиях потребуется всего 340 вольт потенциала от одной моей руки к другой, чтобы вызвать ток 20 миллиампер.Тем не менее, все же возможно получить смертельный удар от меньшего напряжения, чем это. При условии значительно более низкого сопротивления тела, увеличенного за счет контакта с кольцом (полоса золота, обернутая по окружности пальца, является отличной точкой контакта для поражения электрическим током) или полного контакта с большим металлическим предметом, таким как труба или металл рукоятки инструмента, сопротивление корпуса может упасть до 1000 Ом (1 кОм), в результате чего даже более низкое напряжение может представлять потенциальную опасность.

[латекс] E = IR [/ латекс]

[латекс] E = (20 мА) (1 к \ Омега) [/ латекс]

[латекс] \ textbf {E = 20 V} [/ латекс]

Обратите внимание, что в этом состоянии 20 вольт достаточно, чтобы произвести ток в 20 миллиампер через человека; достаточно, чтобы вызвать столбняк. Помните, было высказано предположение, что сила тока всего 17 миллиампер может вызвать фибрилляцию желудочков (сердца). При сопротивлении рукопашной в 1000 Ом для создания этого опасного состояния потребуется всего 17 вольт.

[латекс] E = IR [/ латекс]
[латекс] E = (17 мА) (1 кВт) [/ латекс]
[латекс] \ textbf {E = 17 В} [/ латекс]

Семнадцать вольт — это не очень много для электрических систем. Конечно, это «наихудший» сценарий с напряжением переменного тока 60 Гц и отличной проводимостью тела, но он действительно показывает, насколько низкое напряжение может представлять серьезную угрозу при определенных условиях.

Условия, необходимые для создания сопротивления тела 1000 Ом, не должны быть такими экстремальными, как то, что было представлено (потная кожа при контакте с золотым кольцом).Сопротивление тела может уменьшаться при приложении напряжения (особенно если столбняк заставляет пострадавшего крепче держать проводник), так что при постоянном напряжении удар может усилиться после первого контакта. То, что начинается как легкий шок — ровно настолько, чтобы «заморозить» жертву, чтобы она не могла отпустить ее, может перерасти в нечто достаточно серьезное, чтобы убить ее, поскольку сопротивление их тела уменьшается, а сила тока соответственно увеличивается.

Research предоставило примерный набор цифр для электрического сопротивления точек контакта человека в различных условиях:

Ситуация Сухой Мокрая
Проволока касалась пальцем 40 000 Ом — 1 000 000 Ом 4000 Ом — 15000 Ом
Проволока в руке 15000 Ом — 50 000 Ом 3000 Ом — 5000 Ом
Ручные плоскогубцы по металлу 5000 Ом — 10 000 Ом 1000 Ом — 3000 Ом
Контакт ладонью 3000 Ом — 8000 Ом 1000 Ом — 2000 Ом
1.5-дюймовая металлическая труба с захватом одной рукой 1000 Ом — 3000 Ом 500 Ом — 1500 Ом
Металлическая труба 1,5 дюйма, удерживаемая двумя руками 500 Ом — 1500 кОм 250 Ом — 750 Ом
Рука погружена в проводящую жидкость 200 Ом — 500 Ом
Нога погружена в проводящую жидкость100 Ом — 300 Ом

Обратите внимание на значения сопротивления для двух условий, включающих 1.5-дюймовая металлическая труба. Сопротивление, измеренное при захвате трубы двумя руками, составляет ровно половину сопротивления, когда одна рука держит трубу.

Рисунок 1.8

Двумя руками площадь контакта с телом вдвое больше, чем с одной рукой. Это важный урок: электрическое сопротивление между любыми контактирующими объектами уменьшается с увеличением площади контакта при прочих равных условиях. Если держать трубу двумя руками, ток будет иметь два параллельных пути, по которым протекает от трубы к телу (или наоборот).

Рис. 1.9

Как мы увидим в более поздней главе, параллельных пути цепи всегда приводят к меньшему общему сопротивлению, чем любой отдельный путь, рассматриваемый отдельно.

В промышленности 30 вольт обычно считается консервативным пороговым значением для опасного напряжения. Осторожный человек должен рассматривать любое напряжение выше 30 вольт как опасное, не полагаясь на нормальное сопротивление тела для защиты от поражения электрическим током. Тем не менее, при работе с электричеством все же отличной идеей является держать руки чистыми и сухими и снимать все металлические украшения.Даже при более низком напряжении металлические украшения могут представлять опасность, поскольку проводят ток, достаточный для ожога кожи, при контакте между двумя точками в цепи. Металлические кольца, в частности, были причиной более чем нескольких ожогов пальцев из-за замыкания между точками в низковольтной и сильноточной цепи.

Кроме того, напряжение ниже 30 может быть опасным, если его достаточно, чтобы вызвать неприятное ощущение, которое может вызвать вздрагивание и случайное соприкосновение с более высоким напряжением или другую опасность.Я вспоминаю, как однажды жарким летним днем ​​работал над автомобилем. На мне были шорты, моя голая нога касалась хромового бампера автомобиля, когда я затягивал контакты аккумулятора. Когда я прикоснулся металлическим ключом к положительной (незаземленной) стороне 12-вольтовой батареи, я почувствовал покалывание в том месте, где моя нога касалась бампера. Сочетание плотного контакта с металлом и моей вспотевшей кожи позволило почувствовать шок всего лишь при напряжении 12 вольт.

К счастью, ничего плохого не произошло, но если бы двигатель работал и удар ощущался в моей руке, а не ноге, я мог бы рефлекторно толкнуть руку на пути вращающегося вентилятора или уронить металлический ключ на клеммы аккумулятора (производя большой ток через гаечный ключ с большим количеством сопутствующих искр).Это иллюстрирует еще один важный урок, касающийся электробезопасности; этот электрический ток сам по себе может быть косвенной причиной травмы, заставляя вас подпрыгивать или спазмировать части вашего тела в опасную для вас сторону.

Ток, проходящий через человеческое тело, имеет значение, насколько он опасен. Ток будет влиять на все мышцы, встречающиеся на его пути, а поскольку мышцы сердца и легких (диафрагмы), вероятно, являются наиболее важными для выживания, токи, проходящие через грудную клетку, являются наиболее опасными.Это делает путь электрического тока из рук в руки очень вероятным способом получения травм и смертельного исхода.

Во избежание подобных ситуаций рекомендуется работать с цепями под напряжением, находящимися под напряжением, только одной рукой, а вторую руку держать в кармане, чтобы случайно ни к чему не прикоснуться. Конечно, всегда безопаснее работать в цепи, когда она отключена, но это не всегда практично или возможно. При работе одной рукой, как правило, предпочтение отдается правой руке по двум причинам: большинство людей правши (что обеспечивает дополнительную координацию при работе), а сердце обычно находится слева от центра в грудной полости.

Для левшей этот совет может быть не лучшим. Если такой человек недостаточно скоординирован с правой рукой, он может подвергнуть себя большей опасности, используя ту руку, с которой ему меньше всего комфортно, даже если электрический ток, протекающий через эту руку, может представлять большую опасность для его сердца. Относительная опасность между сотрясением одной рукой или другой, вероятно, меньше, чем опасность работы с менее чем оптимальной координацией, поэтому выбор руки для работы лучше всего оставить на усмотрение человека.

Лучшая защита от ударов цепи под напряжением — это сопротивление, а сопротивление может быть добавлено к телу с помощью изолированных инструментов, перчаток, обуви и другого снаряжения. Ток в цепи является функцией доступного напряжения, деленного на общее сопротивление на пути потока. Как мы рассмотрим более подробно позже в этой книге, сопротивления имеют аддитивный эффект, когда они сложены таким образом, что есть только один путь для прохождения тока:

Рисунок 1.10

Человек, находящийся в прямом контакте с источником напряжения: ток ограничен только сопротивлением тела.

[латекс] I = \ frac {E} {R_ {boot}} [/ латекс]

Теперь мы рассмотрим эквивалентную схему для человека в изолированных перчатках и ботинках:

Рисунок 1.11

Лицо в изоляционных перчатках и сапогах;

Ток теперь ограничен сопротивлением цепи:

[латекс] I = \ frac {E} {R_ {glove} + R_ {body} + R_ {boot} +} [/ latex]

Поскольку электрический ток должен проходить через ботинок и корпус и перчатку, чтобы замкнуть цепь обратно к батарее, общая сумма ( сумма ) этих сопротивлений противодействует протеканию тока в большей степени, чем любое другое. сопротивлений рассматривается индивидуально.

Безопасность — одна из причин, по которой электрические провода обычно покрывают пластиковой или резиновой изоляцией: чтобы значительно увеличить сопротивление между проводником и тем или иным предметом, который может с ним контактировать. К сожалению, было бы слишком дорого изолировать проводники линии электропередач из-за недостаточной изоляции для обеспечения безопасности в случае случайного контакта. Таким образом, безопасность обеспечивается за счет того, что эти стропы должны находиться достаточно далеко вне досягаемости, чтобы никто не мог случайно прикоснуться к ним.

Если возможно, отключите питание цепи перед выполнением каких-либо работ с ней.Вы должны обезопасить все источники вредной энергии, прежде чем систему можно будет считать безопасной для работы. В промышленности обеспечение безопасности цепи, устройства или системы в этом состоянии обычно называют переводом в состояние с нулевой энергией . В центре внимания этого урока, конечно же, электробезопасность. Однако многие из этих принципов применимы и к неэлектрическим системам.

  • Вред для тела зависит от силы электрического тока. Более высокое напряжение позволяет производить более высокие и опасные токи.Сопротивление противостоит току, поэтому высокое сопротивление является хорошей защитой от ударов.
  • Обычно считается, что любое напряжение выше 30 может создавать опасные ударные токи. Металлические украшения определенно плохо носить при работе с электрическими цепями. Кольца, ремешки для часов, ожерелья, браслеты и другие подобные украшения обеспечивают отличный электрический контакт с вашим телом и сами могут проводить ток, достаточный для возникновения ожогов кожи даже при низком напряжении.
  • Низкое напряжение может быть опасным, даже если оно слишком низкое, чтобы напрямую вызвать поражение электрическим током.Их может быть достаточно, чтобы напугать жертву, заставив ее отпрянуть и коснуться чего-то более опасного в непосредственной близости.
  • Когда необходимо работать в «живой» цепи, лучше всего выполнять работу одной рукой, чтобы предотвратить смертельный путь электрического тока из рук в руки (через грудную клетку).
  • Если возможно, отключите питание цепи перед выполнением каких-либо работ с ней.

При работе с оборудованием отключите все источники питания перед выполнением любых работ.В промышленности удаление этих источников питания из схемы, устройства или системы обычно называется переводом в состояние с нулевой энергией . В центре внимания этого урока, конечно же, электробезопасность. Однако многие из этих принципов применимы и к неэлектрическим системам.

Обеспечение безопасности чего-либо в состоянии нулевой энергии означает избавление от любого вида потенциальной или накопленной энергии, включая, помимо прочего:

  • Опасное напряжение
  • Давление пружины
  • Гидравлическое давление (жидкость)
  • Пневматическое (воздушное) давление
  • Подвесной
  • Химическая энергия (легковоспламеняющиеся или иным образом реагирующие вещества)
  • Ядерная энергия (радиоактивные или делящиеся вещества)

Напряжение по своей природе является проявлением потенциальной энергии.В первой главе я даже использовал приподнятую жидкость в качестве аналогии для потенциальной энергии напряжения, имеющей способность (потенциал) производить ток (поток), но не обязательно осознавая этот потенциал, пока не будет установлен подходящий путь для потока. и сопротивление потоку преодолевается. Пара проводов с высоким напряжением между ними не выглядит и не звучит опасно, даже если они несут между собой достаточно потенциальной энергии, чтобы протолкнуть смертоносное количество тока через ваше тело. Несмотря на то, что это напряжение в настоящее время ничего не делает, у него есть потенциал, и этот потенциал необходимо нейтрализовать, прежде чем можно будет безопасно физически контактировать с этими проводами.

Все правильно спроектированные схемы имеют механизмы отключения для снятия напряжения в цепи. Иногда эти «разъединения» служат двойной цели: автоматически размыкаются в условиях чрезмерного тока, и в этом случае мы называем их «автоматическими выключателями». В других случаях выключатели-разъединители представляют собой устройства с ручным управлением без автоматической функции. В любом случае они существуют для вашей защиты и должны использоваться должным образом. Обратите внимание, что устройство отключения должно быть отдельно от обычного выключателя, используемого для включения и выключения устройства.Это предохранительный выключатель, который должен использоваться только для защиты системы в состоянии нулевого потребления энергии:

Рисунок 1.12

Когда выключатель находится в «разомкнутом» положении, как показано (нет непрерывности), цепь разомкнута, и ток не будет. На нагрузке будет нулевое напряжение, а полное напряжение источника будет падать на разомкнутые контакты выключателя. Обратите внимание, что в нижнем проводе цепи нет необходимости в размыкающем выключателе. Поскольку эта сторона цепи надежно соединена с землей (землей), она электрически является общей с землей, и ее лучше оставить таким образом.Для максимальной безопасности персонала, работающего с нагрузкой этой цепи, можно установить временное заземление на верхней стороне нагрузки, чтобы исключить падение напряжения на нагрузке:

Рисунок 1.13

При наличии временного заземляющего соединения обе стороны проводки нагрузки соединяются с землей, обеспечивая нулевое состояние энергии на нагрузке.

Поскольку заземление, выполненное с обеих сторон нагрузки, электрически эквивалентно короткому замыканию через нагрузку с помощью провода, это еще один способ достижения той же цели максимальной безопасности:

Рисунок 1.14

В любом случае обе стороны нагрузки будут электрически общими с землей, с учетом отсутствия напряжения (потенциальной энергии) между обеими сторонами нагрузки и землей, на которой стоят люди. Этот метод временного заземления проводов в обесточенной энергосистеме очень распространен при работах по техническому обслуживанию, выполняемых в системах распределения электроэнергии высокого напряжения.

Еще одним преимуществом этой меры предосторожности является защита от возможности включения размыкающего переключателя (включения, чтобы обеспечить непрерывность цепи), когда люди все еще контактируют с нагрузкой.Временный провод, подключенный к нагрузке, создавал бы короткое замыкание, когда выключатель был замкнут, немедленно отключая любые устройства защиты от перегрузки по току (автоматические выключатели или предохранители) в цепи, что снова отключает питание. Если это произойдет, разъединитель вполне может получить повреждение, но рабочие на нагрузке находятся в безопасности.

Здесь было бы хорошо упомянуть, что устройства максимального тока не предназначены для защиты от поражения электрическим током.Скорее, они существуют исключительно для защиты проводников от перегрева из-за чрезмерных токов. Только что описанные временные закорачивающие провода действительно могут вызвать «срабатывание» любых устройств перегрузки по току в цепи, если выключатель должен быть замкнут, но следует понимать, что защита от поражения электрическим током не является предполагаемой функцией этих устройств. Их основная функция будет просто использоваться для защиты рабочего с установленным закорачивающим проводом.

Структурированные системы безопасности: блокировка / маркировка

Поскольку очевидно, что важно иметь возможность закрепить любые отключающие устройства в разомкнутом (выключенном) положении и убедиться, что они остаются в этом положении во время работы в цепи, существует потребность в структурированной системе безопасности, которая должна быть введена в место.Такая система обычно используется в промышленности и называется Lock-out / Tag-out .

Процедура блокировки / маркировки работает следующим образом: все люди, работающие в защищенной цепи, имеют свой собственный замок или кодовый замок, который они устанавливают на рычаге управления устройства отключения перед работой с системой. Кроме того, они должны заполнить и подписать ярлык, который они вешают на замок, с описанием характера и продолжительности работы, которую они собираются выполнять в системе.Если есть несколько источников энергии, которые необходимо «заблокировать» (множественные разъединения, как электрические, так и механические источники энергии должны быть защищены, и т. Д.), Рабочий должен использовать столько своих замков, сколько необходимо для обеспечения питания от системы. до начала работы. Таким образом, система поддерживается в состоянии нулевого энергопотребления до тех пор, пока не будет снята каждая последняя блокировка со всех устройств отключения и отключения, а это означает, что каждый последний работник даст согласие, сняв свои личные блокировки. Если было принято решение повторно активировать систему, и замок (и) одного человека все еще остается на месте после того, как все присутствующие снимают свои, метка (и) покажет, кто этот человек и что он делает.

Даже при наличии хорошей программы безопасности по блокировке / маркировке все еще необходимы усердие и меры предосторожности, основанные на здравом смысле. Это особенно актуально в промышленных условиях, когда над устройством или системой может одновременно работать множество людей. Некоторые из этих людей могут не знать о надлежащей процедуре блокировки / маркировки или могут знать о ней, но слишком самоуверенны, чтобы ей следовать. Не думайте, что все соблюдают правила безопасности!

После того, как электрическая система была заблокирована и помечена вашим личным замком, вы должны дважды проверить, действительно ли напряжение было зафиксировано в нулевом состоянии.Один из способов проверить — увидеть, запустится ли машина (или что-то еще, над чем она работает), если будет задействован выключатель или кнопка start . Если он запускается, значит, вы знаете, что не смогли обеспечить от него электрическую энергию.

Кроме того, вы должны всегда проверять на наличие опасного напряжения с помощью измерительного прибора, прежде чем касаться каких-либо проводников в цепи. Для большей безопасности вы должны выполнить следующую процедуру проверки, использования, а затем проверки вашего глюкометра:

  • Убедитесь, что ваш измеритель правильно показывает на известном источнике напряжения.
  • Используйте свой измеритель, чтобы проверить цепь блокировки на наличие опасного напряжения.
  • Еще раз проверьте свой измеритель на известном источнике напряжения, чтобы убедиться, что он по-прежнему показывает, как должен.

Хотя это может показаться чрезмерным или даже параноидальным, это проверенный метод предотвращения поражения электрическим током. Однажды у меня был счетчик, который не смог показать напряжение, когда он должен был, при проверке цепи, чтобы убедиться, что она «мертвая». Если бы я не использовал другие средства для проверки наличия напряжения, меня бы сегодня не было в живых, чтобы написать это.Всегда есть шанс, что ваш вольтметр окажется неисправным именно тогда, когда он понадобится вам для проверки на наличие опасного состояния. Следуя этим инструкциям, вы никогда не попадете в смертельную ситуацию из-за поломки счетчика.

Наконец, электротехник прибудет к тому моменту процедуры проверки безопасности, когда будет считаться безопасным прикосновение к проводнику (проводам). Имейте в виду, что после принятия всех мер предосторожности возможно (хотя и очень маловероятно) наличие опасного напряжения.Последней мерой предосторожности, которую следует предпринять на этом этапе, является кратковременный контакт проводника (проводов) тыльной стороной руки перед тем, как схватить его или металлический инструмент, соприкасающийся с ним. Почему? Если по какой-то причине напряжение между этим проводником и землей все еще присутствует, движение пальца в результате реакции удара (сжатие в кулак) приведет к разрыву контакта с проводом. Обратите внимание, что это абсолютно последний шаг , который должен выполнить любой электромонтер перед началом работы с энергосистемой, и не следует использовать никогда не в качестве альтернативного метода проверки опасного напряжения.Если у вас когда-либо будут основания сомневаться в надежности вашего глюкометра, воспользуйтесь другим глюкометром, чтобы получить «второе мнение».

  • Состояние нулевой энергии: когда цепь, устройство или система защищены таким образом, что отсутствует потенциальная энергия, которая могла бы нанести вред кому-либо, работающему с ними.
  • Устройства выключателя
  • должны присутствовать в правильно спроектированной электрической системе, чтобы обеспечить удобную готовность к состоянию нулевого потребления энергии.
  • К обслуживаемой нагрузке могут быть подключены временные заземляющие или закорачивающие провода для дополнительной защиты персонала, работающего с этой нагрузкой.
  • Lock-out / Tag-out работает следующим образом: при работе с системой в состоянии нулевого энергопотребления рабочий помещает личный замок или кодовый замок на каждое устройство отключения энергии, имеющее отношение к его или ее задаче в этой системе. Кроме того, на каждый из этих замков навешивается тег, описывающий характер и продолжительность работы, которую необходимо выполнить, и того, кто ее выполняет.
  • Всегда проверяйте, что цепь была зафиксирована в состоянии нулевого энергопотребления с помощью испытательного оборудования после «блокировки». Обязательно проверьте свой глюкометр до и после проверки цепи, чтобы убедиться, что она работает правильно.
  • Когда придет время действительно вступить в контакт с проводником (-ами) предположительно мертвой энергосистемы, сделайте это сначала тыльной стороной одной руки, чтобы в случае удара током мышечная реакция оттолкнула пальцы от проводника. .

Безопасное и эффективное использование электрического счетчика — это, пожалуй, самый ценный навык, которым может овладеть электронщик, как ради собственной безопасности, так и для профессионального мастерства. Поначалу может быть сложно использовать счетчик, зная, что вы подключаете его к цепям под напряжением, которые могут содержать опасные для жизни уровни напряжения и тока.Это опасение небезосновательно, и всегда лучше действовать осторожно при использовании счетчиков. Небрежность больше, чем какой-либо другой фактор, является причиной несчастных случаев с электричеством у опытных технических специалистов.

Мультиметры

Самым распространенным электрическим испытательным оборудованием является мультиметр . Мультиметры названы так потому, что они могут измерять множество переменных: напряжение, ток, сопротивление и часто многие другие, некоторые из которых не могут быть описаны здесь из-за их сложности.В руках обученного техника мультиметр является одновременно эффективным рабочим инструментом и защитным устройством. Однако в руках невежественного и / или неосторожного человека мультиметр может стать источником опасности при подключении к «действующей» цепи.

Существует много разных марок мультиметров, причем каждый производитель выпускает несколько моделей с разными наборами функций. Мультиметр, показанный здесь на следующих иллюстрациях, представляет собой «общую» конструкцию, не специфичную для какого-либо производителя, но достаточно общую, чтобы научить основным принципам использования:

Рисунок 1.15

Вы заметите, что дисплей этого измерителя имеет «цифровой» тип: числовые значения отображаются с использованием четырех цифр, как на цифровых часах. Поворотный селекторный переключатель (теперь установлен в положение Off ) имеет пять различных положений измерения, в которых он может быть установлен: два положения «V», два положения «A» и одно положение посередине с забавной «подковой». Символ на нем, представляющий «сопротивление». Символ «подкова» — это греческая буква «Омега» (Ω), которая является общим символом для электрической единицы измерения Ом.

Из двух настроек «V» и двух настроек «A» вы заметите, что каждая пара разделена на уникальные маркеры либо парой горизонтальных линий (одна сплошная, одна пунктирная), либо пунктирной линией с волнистой кривой над ней. . Параллельные линии представляют «постоянный ток», а волнистая кривая — «переменный ток». «V», конечно, означает «напряжение», а «A» означает «сила тока» (ток). В измерителе для измерения постоянного тока используются другие методы, чем для измерения переменного тока, поэтому пользователю необходимо выбрать тип напряжения (В) или тока (А) для измерения.Хотя мы не обсуждали переменный ток (AC) в каких-либо технических деталях, это различие в настройках счетчика важно помнить.

Мультиметр Розетки

На лицевой панели мультиметра есть три разных гнезда, к которым мы можем подключить наши измерительные провода . Измерительные провода — это не что иное, как специально подготовленные провода, используемые для подключения измерителя к тестируемой цепи. Провода покрыты гибкой изоляцией с цветовой кодировкой (черной или красной), чтобы руки пользователя не касались оголенных проводов, а кончики зондов представляют собой острые жесткие кусочки проволоки:

Рисунок 1.16

Черный измерительный провод всегда вставляется в черный разъем на мультиметре: тот, который отмечен «COM» для «общего». Красные измерительные провода подключаются либо к красному разъему с маркировкой для напряжения и сопротивления, либо к красному разъему с маркировкой для тока, в зависимости от того, какое количество вы собираетесь измерить с помощью мультиметра.

Чтобы увидеть, как это работает, давайте рассмотрим несколько примеров, показывающих, как используется счетчик. Сначала мы настроим измеритель для измерения постоянного напряжения от батареи:

Рисунок 1.17

Обратите внимание, что два измерительных провода подключены к соответствующим гнездам на измерителе напряжения, а селекторный переключатель установлен на «V» постоянного тока. Теперь рассмотрим пример использования мультиметра для измерения напряжения переменного тока от бытовой электрической розетки (настенной розетки):

Рисунок 1.18

Единственное отличие в настройке счетчика — это расположение селекторного переключателя: теперь он установлен на переменный ток «V». Поскольку мы все еще измеряем напряжение, измерительные провода останутся подключенными к тем же гнездам.В обоих этих примерах настоятельно рекомендуется, , чтобы вы не позволяли наконечникам щупов соприкасаться друг с другом, пока они оба находятся в контакте со своими соответствующими точками в цепи. Если это произойдет, произойдет короткое замыкание, создающее искру и, возможно, даже шар пламени, если источник напряжения способен обеспечить достаточный ток! Следующее изображение иллюстрирует потенциальную опасность:

Рис. 1.19.

Это лишь один из способов, которым счетчик может стать источником опасности при неправильном использовании.

Измерение напряжения, пожалуй, самая распространенная функция, для которой используется мультиметр. Это, безусловно, первичное измерение, выполняемое в целях безопасности (часть процедуры блокировки / маркировки), и оно должно быть хорошо понято оператором счетчика. Поскольку напряжение между двумя точками всегда является относительным, измеритель должен быть надежно подключен к двум точкам в цепи, прежде чем он будет обеспечивать надежное измерение. Обычно это означает, что оба щупа должны быть схвачены руками пользователя и прижаты к правильным точкам контакта источника напряжения или цепи во время измерения.

Поскольку путь электрического тока из рук в руки является наиболее опасным, удерживание измерительных щупов в двух точках высоковольтной цепи таким образом всегда представляет собой потенциальную опасность . Если защитная изоляция на датчиках изношена или потрескалась, пальцы пользователя могут соприкоснуться с проводниками датчика во время испытания, что приведет к сильному удару. Если можно использовать только одну руку для захвата зондов, это более безопасный вариант. Иногда можно «защелкнуть» один наконечник щупа на контрольной точке цепи, чтобы его можно было отпустить, а другой установить на место, используя только одну руку.Для облегчения этого можно прикрепить специальные аксессуары для наконечников зонда, такие как пружинные зажимы.

Помните, что измерительные провода измерителя являются частью всего комплекта оборудования и что с ними следует обращаться так же осторожно и уважительно, как и с самим измерителем. Если вам нужен специальный аксессуар для ваших измерительных проводов, такой как пружинный зажим или другой специальный наконечник зонда, обратитесь к каталогу продукции производителя измерителя или другого производителя испытательного оборудования. Не пытайтесь проявить изобретательность и изготавливать свои собственные пробники, так как вы можете подвергнуть себя опасности в следующий раз, когда будете использовать их в цепи под напряжением.

Также следует помнить, что цифровые мультиметры обычно хорошо справляются с различением измерений переменного и постоянного тока, поскольку они настраиваются на одно или другое при проверке напряжения или тока. Как мы видели ранее, как переменное, так и постоянное напряжение и ток могут быть смертельными, поэтому при использовании мультиметра в качестве устройства проверки безопасности вы всегда должны проверять наличие как переменного, так и постоянного тока, даже если вы не ожидаете найти и то, и другое. ! Кроме того, при проверке наличия опасного напряжения вы должны обязательно проверить всех пар точек, о которых идет речь.

Например, предположим, что вы открыли шкаф с электропроводкой и обнаружили три больших проводника, подающих питание переменного тока на нагрузку. Автоматический выключатель, питающий эти провода (предположительно), был отключен, заблокирован и помечен. Вы дважды проверили отсутствие питания, нажав кнопку Start для нагрузки. Ничего не произошло, поэтому теперь вы переходите к третьему этапу проверки безопасности: проверке измерителя напряжения.

Сначала вы проверяете свой измеритель на известном источнике напряжения, чтобы убедиться, что он работает правильно.Любая ближайшая электрическая розетка должна обеспечивать удобный источник переменного напряжения для проверки. Вы делаете это и обнаруживаете, что счетчик показывает как следует. Затем вам нужно проверить напряжение между этими тремя проводами в шкафу. Но напряжение измеряется между двумя точками , так где же проверить?

Рис. 1.20

Ответ — проверить все комбинации этих трех точек. Как видите, на рисунке точки обозначены буквами «A», «B» и «C», поэтому вам нужно будет взять мультиметр (установленный в режиме вольтметра) и проверить его между точками A и B, B и C, а также A и C.Если вы обнаружите напряжение между любой из этих пар, цепь не находится в состоянии нулевой энергии. Но ждать! Помните, что мультиметр не будет регистрировать напряжение постоянного тока, когда он находится в режиме переменного напряжения, и наоборот, поэтому вам необходимо проверить эти три пары точек в в каждом режиме , в общей сложности шесть проверок напряжения для завершения!

Однако, даже несмотря на всю эту проверку, мы еще не охватили все возможности. Помните, что опасное напряжение может появиться между одиночным проводом и землей (в этом случае металлический каркас шкафа будет хорошей точкой отсчета заземления) в энергосистеме.Итак, чтобы быть в полной безопасности, мы должны не только проверять между A и B, B и C, и A и C (как в режимах переменного, так и постоянного тока), но мы также должны проверять между A и землей, B и землей, и C и заземление (как в режимах переменного, так и постоянного тока)! Это дает в общей сложности двенадцать проверок напряжения для этого, казалось бы, простого сценария всего с тремя проводами. Затем, конечно же, после того, как мы завершили все эти проверки, нам нужно взять мультиметр и повторно проверить его с помощью известного источника напряжения, такого как розетка, чтобы убедиться, что он по-прежнему в хорошем рабочем состоянии.

Использование мультиметра для проверки сопротивления

Использование мультиметра для проверки сопротивления — гораздо более простая задача. Измерительные провода будут оставаться подключенными к тем же розеткам, что и для проверки напряжения, но селекторный переключатель необходимо повернуть, пока он не укажет на символ сопротивления «подкова». Касаясь щупами устройства, сопротивление которого необходимо измерить, измеритель должен правильно отображать сопротивление в омах:

Рисунок 1.21

При измерении сопротивления следует помнить, что это должно выполняться только на обесточенных компонентах ! Когда измеритель находится в режиме «сопротивления», он использует небольшую внутреннюю батарею для генерации крошечного тока через измеряемый компонент. Путем определения того, насколько сложно пропустить этот ток через компонент, можно определить и отобразить сопротивление этого компонента. Если в контуре измерителя-вывод-компонент-вывод-измеритель имеется дополнительный источник напряжения, который либо помогает, либо противодействует току измерения сопротивления, производимому измерителем, это приведет к ошибочным показаниям.В худшем случае счетчик может даже выйти из строя из-за внешнего напряжения.

Режим «Сопротивление» мультиметра

Режим «сопротивления» мультиметра очень полезен для определения целостности проводов, а также для точных измерений сопротивления. Когда между наконечниками пробников имеется хорошее, прочное соединение (моделируется путем их соприкосновения), измеритель показывает почти нулевое сопротивление. Если бы измерительные провода не имели сопротивления, он показывал бы ровно ноль:

. Рисунок 1.22

Если выводы не соприкасаются друг с другом или не касаются противоположных концов разорванного провода, измеритель покажет бесконечное сопротивление (обычно путем отображения пунктирных линий или сокращения «O.L.», что означает «разомкнутый контур»):

Рисунок 1.23

Измерение тока с помощью мультиметра

Безусловно, наиболее опасным и сложным применением мультиметра является измерение тока. Причина этого довольно проста: для того, чтобы измеритель мог измерять ток, измеряемый ток должен проходить через счетчика.Это означает, что измеритель должен быть частью цепи тока, а не просто подключаться к какой-либо стороне, как в случае измерения напряжения. Чтобы сделать счетчик частью пути тока цепи, исходная цепь должна быть «разорвана», а счетчик соединен через две точки разомкнутого разрыва. Чтобы настроить измеритель на это, переключатель должен указывать на переменный или постоянный ток «A», а красный измерительный провод должен быть вставлен в красную розетку с маркировкой «A». На следующем рисунке показан измеритель, полностью готовый к измерению тока, и проверяемая цепь:

Рисунок 1.24

Сейчас цепь разомкнута при подготовке к подключению счетчика:

Рисунок 1.25

Следующий шаг — вставить измеритель в линию со схемой, подключив два наконечника щупа к разомкнутым концам цепи, черный щуп к отрицательной (-) клемме 9-вольтовой батареи и красный щуп к свободному концу провода, ведущему к лампе:

Рисунок 1.26

Этот пример показывает очень безопасную схему для работы. Напряжение 9 вольт вряд ли представляет опасность поражения электрическим током, поэтому не стоит бояться разомкнуть эту цепь (не голыми руками, не меньше!) И подключить счетчик параллельно с током.Однако с цепями более высокой мощности это действительно может быть опасным занятием. Даже если напряжение в цепи было низким, нормальный ток мог быть достаточно высоким, чтобы возникла опасная искра в момент установления последнего подключения датчика измерителя.

Другой потенциальной опасностью использования мультиметра в режиме измерения тока («амперметр») является невозможность правильно вернуть его в конфигурацию измерения напряжения перед измерением напряжения с его помощью. Причины этого зависят от конструкции и работы амперметра.При измерении тока в цепи путем размещения измерителя непосредственно на пути тока, лучше всего, чтобы измеритель оказывал небольшое сопротивление току или не оказывал никакого сопротивления. В противном случае дополнительное сопротивление изменит работу схемы. Таким образом, мультиметр спроектирован так, чтобы сопротивление между наконечниками измерительного щупа было практически нулевым, когда красный щуп был вставлен в красное гнездо «А» (для измерения тока). В режиме измерения напряжения (красный провод вставлен в красное гнездо «V») между наконечниками измерительных щупов имеется большое количество мегаомов сопротивления, поскольку вольтметры рассчитаны на сопротивление, близкое к бесконечному (так что они не работают). t потреблять значительный ток от тестируемой цепи).

При переключении мультиметра из режима измерения тока в режим измерения напряжения легко повернуть селекторный переключатель из положения «A» в положение «V» и забыть, соответственно, переключить положение разъема красного измерительного провода с «A» на положение «V». «V». В результате — если счетчик затем подключить к источнику значительного напряжения — произойдет короткое замыкание счетчика!

Рисунок 1.27

Чтобы предотвратить это, большинство мультиметров имеют функцию предупреждения, которая издает звуковой сигнал, если когда-либо в гнездо «A» вставлен провод, а селекторный переключатель установлен в положение «V».Однако какими бы удобными ни были эти функции, они по-прежнему не заменяют ясного мышления и осторожности при использовании мультиметра.

Все качественные мультиметры содержат внутри предохранители, которые спроектированы так, чтобы «перегорать» в случае чрезмерного тока через них, как в случае, показанном на последнем изображении. Как и все устройства максимальной токовой защиты, эти предохранители предназначены в первую очередь для защиты оборудования (в данном случае самого счетчика) от чрезмерного повреждения и только во вторую очередь для защиты пользователя от повреждений.Мультиметр можно использовать для проверки собственного предохранителя, установив селекторный переключатель в положение сопротивления и создав соединение между двумя красными гнездами следующим образом:

Рисунок 1.28.

. Исправный предохранитель будет указывать на очень низкое сопротивление, в то время как перегоревший предохранитель всегда будет показывать «O.L.» (или любое другое указание, которое эта модель мультиметра использует для обозначения отсутствия непрерывности). Фактическое количество Ом, отображаемое для исправного предохранителя, не имеет большого значения, пока оно является произвольно низким.

Итак, теперь, когда мы увидели, как использовать мультиметр для измерения напряжения, сопротивления и тока, что еще нужно знать? Множество! Ценность и возможности этого универсального испытательного прибора станут более очевидными по мере того, как вы приобретете навыки и познакомитесь с ним.Ничто не заменит регулярные занятия со сложными инструментами, такими как эти, поэтому не стесняйтесь экспериментировать с безопасными схемами с батарейным питанием.

  • Измеритель, способный проверять напряжение, ток и сопротивление, называется мультиметром .
  • Поскольку напряжение между двумя точками всегда относительное, измеритель напряжения («вольтметр») должен быть подключен к двум точкам в цепи, чтобы получить хорошие показания. Будьте осторожны, не касайтесь оголенных наконечников щупов вместе при измерении напряжения, так как это приведет к короткому замыканию!
  • Не забывайте всегда проверять как напряжение переменного, так и постоянного тока при использовании мультиметра для проверки наличия опасного напряжения в цепи.Убедитесь, что вы проверяете напряжение между всеми комбинациями пар проводников, в том числе между отдельными проводниками и землей!
  • В режиме измерения напряжения («вольтметр») мультиметры имеют очень высокое сопротивление между выводами.
  • Никогда не пытайтесь измерить сопротивление или целостность цепи с помощью мультиметра в цепи, которая находится под напряжением. В лучшем случае показания сопротивления, которые вы получаете от глюкометра, будут неточными, а в худшем случае глюкометр может быть поврежден, а вы можете получить травму.
  • Измерители тока («амперметры») всегда подключены в цепь, поэтому электроны должны проходить через через счетчик.
  • В режиме измерения тока («амперметр») мультиметры практически не имеют сопротивления между выводами. Это сделано для того, чтобы электроны могли проходить через счетчик с наименьшими трудностями. Если бы это было не так, измеритель добавлял бы дополнительное сопротивление в цепи, тем самым влияя на ток.

Как мы видели ранее, энергосистема без надежного заземления непредсказуема с точки зрения безопасности.Невозможно гарантировать, сколько или как мало будет напряжения между любой точкой цепи и землей. Заземлив одну сторону источника напряжения энергосистемы, по крайней мере, одна точка в цепи может быть электрически соединена с землей и, следовательно, не представляет опасности поражения электрическим током. В простой двухпроводной системе электропитания проводник, соединенный с землей, называется нейтраль , а другой провод — hot , также известный как live или active :

Рисунок 1.29 Двухпроводная система электропитания

Что касается источника напряжения и нагрузки, заземление не имеет никакого значения. Он существует исключительно ради личной безопасности, гарантируя, что по крайней мере одна точка в цепи будет безопасна для прикосновения (нулевое напряжение относительно земли). «Горячая» сторона цепи, названная так из-за ее потенциальной опасности поражения электрическим током, будет опасна при прикосновении, если напряжение не будет обеспечено надлежащим отключением от источника (в идеале, с использованием процедуры систематической блокировки / маркировки).

Этот дисбаланс опасностей между двумя проводниками в простой силовой цепи важно понимать. Следующая серия иллюстраций основана на распространенных бытовых системах электропроводки (для простоты используются источники постоянного напряжения, а не переменного тока).

Если мы посмотрим на простой бытовой электроприбор, такой как тостер с проводящим металлическим корпусом, мы увидим, что при правильной работе не должно быть опасности поражения электрическим током. Провода, подающие питание на нагревательные элементы тостера, изолированы от соприкосновения с металлическим корпусом (и друг с другом) резиной или пластиком.

Рисунок 1.30 Отсутствие напряжения между корпусом и землей

Однако, если один из проводов внутри тостера случайно войдет в контакт с металлическим корпусом, корпус станет электрически общим для провода, и прикосновение к корпусу будет столь же опасным, как прикосновение к оголенному проводу. Представляет ли это опасность поражения электрическим током, зависит от номера , к которому случайно задевает провод :

Рисунок 1.31 случайное контактное напряжение между корпусом и землей

Если «горячий» провод касается корпуса, это подвергает опасности пользователя тостера.С другой стороны, если нейтральный провод касается корпуса, опасности поражения электрическим током нет:

Рисунок 1.32 Случайное отсутствие напряжения между корпусом и землей

Чтобы гарантировать, что первый отказ менее вероятен, чем второй, инженеры стараются проектировать устройства таким образом, чтобы свести к минимуму контакт горячего проводника с корпусом. В идеале, конечно, вы не хотите, чтобы какой-либо из проводов случайно соприкасался с проводящим корпусом прибора, но обычно есть способы спроектировать расположение частей, чтобы сделать случайный контакт менее вероятным для одного провода, чем для другого.

Однако эта профилактическая мера эффективна только в том случае, если может быть гарантирована полярность вилки питания. Если вилку можно перевернуть, то проводник с большей вероятностью соприкоснется с корпусом вполне может быть «горячим»:

Рисунок 1.33 Напряжение между корпусом и землей

Устройства, разработанные таким образом, обычно поставляются с «поляризованными» вилками, причем один контакт вилки немного уже, чем другой. Розетки питания также имеют такую ​​же конструкцию, причем один слот уже другой.Следовательно, вилку нельзя вставить «задом наперед», и можно гарантировать идентичность проводника внутри устройства. Помните, что это никак не влияет на основные функции устройства: это делается исключительно ради безопасности пользователя.

Некоторые инженеры решают проблему безопасности, просто делая внешний корпус прибора непроводящим. Такие приборы называются с двойной изоляцией, , поскольку изолирующий кожух служит вторым слоем изоляции над и за пределами самих проводов.Если провод внутри устройства случайно войдет в контакт с корпусом, это не представляет опасности для пользователя устройства.

Другие инженеры решают проблему безопасности, поддерживая проводящий корпус, но используя третий провод для надежного соединения этого корпуса с землей:

Рис. 1.34 Нулевое напряжение корпуса заземления между корпусом и землей

Третий контакт на шнуре питания обеспечивает прямое электрическое соединение корпуса устройства с землей, делая две точки электрически общими друг с другом.Если они электрически общие, то между ними не может быть падения напряжения. По крайней мере, так оно и должно работать. Если горячий провод случайно коснется металлического корпуса прибора, он вызовет прямое короткое замыкание обратно на источник напряжения через заземляющий провод, сработав любые устройства защиты от сверхтока. Пользователь устройства останется в безопасности.

Вот почему так важно никогда не отрезать третий контакт вилки питания, когда пытаетесь вставить его в розетку с двумя контактами.Если это будет сделано, не будет заземления корпуса прибора для обеспечения безопасности пользователя (ей). Устройство по-прежнему будет функционировать должным образом, но в случае внутренней неисправности, приводящей к контакту горячей проволоки с корпусом, результаты могут быть смертельными. Если необходимо использовать двухконтактную розетку , можно установить двухконтактный переходник розетки с заземляющим проводом, прикрепленным к винту заземляющей крышки. Это обеспечит безопасность заземленного прибора, подключенного к розетке этого типа.

Однако электрическая безопасность не обязательно заканчивается нагрузкой. Последнюю защиту от поражения электрическим током можно установить на стороне источника питания цепи, а не на самом приборе. Эта мера защиты называется , обнаружение замыкания на землю , и работает она следующим образом:

В правильно работающем приборе (показанном выше) ток, измеренный через проводник под напряжением, должен быть точно равен току через нейтральный проводник, потому что существует только один путь для прохождения электронов в цепи.При отсутствии неисправности внутри устройства нет соединения между проводниками цепи и человеком, касающимся корпуса, и, следовательно, нет удара.

Если, однако, горячая проволока случайно коснется металлического корпуса, через человека, прикоснувшегося к корпусу, пройдет ток. Наличие электрического тока будет проявляться как разница тока между двумя силовыми проводниками в розетке:

Рисунок 1.35 Разница в токе между двумя силовыми проводниками в розетке

Эта разница в токе между «горячим» и «нейтральным» проводниками будет существовать только в том случае, если есть ток через заземление, что означает, что в системе есть неисправность.Следовательно, такая разница тока может использоваться как способ обнаружения неисправного состояния. Если устройство настроено для измерения этой разницы в токах между двумя силовыми проводниками, обнаружение дисбаланса тока можно использовать для запуска размыкания выключателя, тем самым отключая питание и предотвращая серьезный удар:

Рисунок 1.36 Прерыватели тока замыкания на землю

Такие устройства называются Прерыватели тока замыкания на землю , или сокращенно GFCI. За пределами Северной Америки GFCI также известен как предохранительный выключатель, устройство защитного отключения (RCD), RCBO или RCD / MCB в сочетании с миниатюрным автоматическим выключателем или выключателем утечки на землю (ELCB).Они достаточно компактны, чтобы их можно было встроить в розетку. Эти розетки легко идентифицировать по их характерным кнопкам «Тест» и «Сброс». Большим преимуществом использования этого подхода для обеспечения безопасности является то, что он работает независимо от конструкции устройства. Конечно, использование прибора с двойной изоляцией или заземлением в дополнение к розетке GFCI было бы еще лучше, но приятно знать, что можно что-то сделать для повышения безопасности, помимо конструкции и состояния прибора.

Прерыватель цепи дугового замыкания (AFCI) , автоматический выключатель, предназначенный для предотвращения пожаров, предназначен для размыкания при прерывистых резистивных коротких замыканиях. Например, нормальный выключатель на 15 А предназначен для быстрого размыкания цепи при нагрузке, значительно превышающей номинальную 15 А, или медленнее, немного превышающей номинальную. Хотя это защищает от прямого короткого замыкания и нескольких секунд перегрузки, соответственно, он не защищает от дуги — аналогично дуговой сварке. Дуга представляет собой сильно изменяющуюся нагрузку, периодически достигающую максимума более 70 А, разомкнутую цепь с переходами через ноль переменного тока.Хотя среднего тока недостаточно для срабатывания стандартного выключателя, его достаточно, чтобы разжечь пожар. Эта дуга может быть создана из-за металлического короткого замыкания, которое сжигает металл, оставляя резистивную распыляющую плазму ионизированных газов.

AFCI содержит электронную схему для обнаружения этого прерывистого резистивного короткого замыкания. Он защищает как от дуги от горячего к нейтральному, так и от горячего к заземлению. AFCI не защищает от опасности поражения электрическим током, как GFCI. Таким образом, GFCI по-прежнему необходимо устанавливать на кухне, в ванной и на открытом воздухе.Поскольку AFCI часто срабатывает при запуске больших двигателей и, в более общем смысле, щеточных двигателей, его установка ограничена электрическими цепями в спальнях согласно Национальному электротехническому кодексу США. Использование AFCI должно уменьшить количество электрических пожаров. Однако неприятные срабатывания при работе приборов с двигателями в цепях AFCI представляют собой проблему.

  • В энергосистемах одна сторона источника напряжения часто подключается к заземлению для обеспечения безопасности в этой точке.
  • «Заземленный» провод в энергосистеме называется нейтральным проводником , а незаземленный провод — горячим проводом .
  • Заземление в энергосистемах существует для личной безопасности, а не для работы нагрузки (ей).
  • Электробезопасность прибора или других нагрузок может быть улучшена за счет хорошей инженерии: поляризованные вилки, двойная изоляция и трехконтактные вилки с «заземлением» — все это способы повышения безопасности на стороне нагрузки.
  • Прерыватели тока замыкания на землю (GFCI) работают, считывая разницу в токе между двумя проводниками, подающими питание на нагрузку.Никакой разницы в токе быть не должно. Любое различие означает, что ток должен входить в нагрузку или выходить из нее каким-либо образом, кроме двух основных проводников, что нехорошо. Значительная разница в токе автоматически откроет размыкающий механизм выключателя, полностью отключив питание.

Обычно допустимая токовая нагрузка проводника является пределом конструкции схемы, который нельзя намеренно превышать, но есть приложение, в котором ожидается превышение допустимой токовой нагрузки: в случае предохранителей .

Что такое предохранитель?

A Предохранитель представляет собой устройство электробезопасности, построенное вокруг проводящей полосы, которая предназначена для плавления и разделения в случае чрезмерного тока. Предохранители всегда подключаются последовательно с компонентом (ами), который должен быть защищен от перегрузки по току, так что, когда плавкий предохранитель перегорает (размыкается), он размыкает всю цепь и останавливает ток через компонент (ы). Плавкий предохранитель, включенный в одну ветвь параллельной цепи, конечно, не повлияет на ток через любую из других ветвей.

Обычно тонкий кусок плавкой проволоки помещается в защитную оболочку, чтобы свести к минимуму опасность возникновения дугового разряда в случае прорыва проволоки с большой силой, что может произойти в случае сильных перегрузок по току. В случае небольших автомобильных предохранителей оболочка является прозрачной, так что плавкий элемент может быть визуально осмотрен. В бытовой электропроводке обычно используются ввинчиваемые предохранители со стеклянным корпусом и тонкой узкой полосой из металлической фольги посередине. Фотография, показывающая оба типа предохранителей, представлена ​​здесь:

Рисунок 1.37 Типы предохранителей

Предохранители картриджного типа популярны в автомобилях и в промышленности, если они изготовлены из материалов оболочки, отличных от стекла. Поскольку предохранители рассчитаны на «отказ» срабатывания при превышении их номинального тока, они обычно предназначены для легкой замены в цепи. Это означает, что они будут вставлены в какой-либо тип держателя, а не припаиваться или прикрепляться болтами к проводникам схемы. Ниже приведена фотография, на которой изображена пара предохранителей со стеклянным картриджем в держателе с несколькими предохранителями:

Рисунок 1.38 Стеклянный патрон с предохранителями Держатель нескольких предохранителей

Предохранители удерживаются пружинными металлическими зажимами, причем сами зажимы постоянно соединены с проводниками цепи. Основной материал держателя предохранителя (или блока предохранителей , как их иногда называют) выбран как хороший изолятор.

Другой тип держателя предохранителей патронного типа обычно используется для установки в панелях управления оборудованием, где желательно скрыть все точки электрического контакта от контакта с человеком.В отличие от только что показанного блока предохранителей, где все металлические зажимы открыты, этот тип держателя предохранителя полностью закрывает предохранитель в изоляционном корпусе:

Рисунок 1.39 Патрон предохранителя закрывает изолирующий корпус

Наиболее распространенным устройством защиты от перегрузки по току в сильноточных цепях сегодня является автоматический выключатель .

Что такое автоматический выключатель?

Автоматические выключатели — это специально разработанные переключатели, которые автоматически размыкаются для отключения тока в случае перегрузки по току.Малые автоматические выключатели, например, используемые в жилых, коммерческих и легких промышленных предприятиях, имеют термическое управление. Они содержат биметаллическую полосу (тонкую полосу из двух металлов, соединенных спина к спине), несущую ток цепи, которая изгибается при нагревании. Когда биметаллическая полоса создает достаточную силу (из-за чрезмерного нагрева полосы), срабатывает механизм отключения, и прерыватель размыкается. Автоматические выключатели большего размера автоматически активируются силой магнитного поля, создаваемого токонесущими проводниками внутри выключателя, или могут срабатывать для отключения от внешних устройств, контролирующих ток цепи (эти устройства называются защитными реле , ).

Поскольку автоматические выключатели не выходят из строя в условиях перегрузки по току — скорее, они просто размыкаются и могут быть повторно включены путем перемещения рычага — они с большей вероятностью будут обнаружены подключенными к цепи более прочным образом, чем предохранители. Фотография маленького автоматического выключателя представлена ​​здесь:

Рисунок 1.40. Малый автоматический выключатель

Снаружи он выглядит как выключатель. Действительно, его можно было использовать как таковое. Однако его истинная функция — работать как устройство защиты от перегрузки по току.

Следует отметить, что в некоторых автомобилях используются недорогие устройства, известные как плавкие вставки , для защиты от перегрузки по току в цепи зарядки аккумулятора из-за стоимости предохранителя и держателя надлежащего номинала. Плавкая вставка — это примитивный предохранитель, представляющий собой не что иное, как короткий кусок провода с резиновой изоляцией, предназначенный для плавления в случае перегрузки по току, без какой-либо твердой оболочки. Такие грубые и потенциально опасные устройства никогда не используются в промышленности или даже в жилых помещениях, в основном из-за встречающихся более высоких уровней напряжения и тока.По мнению автора, их применение даже в автомобильных схемах вызывает сомнения.

Обозначение предохранителя на принципиальной электрической схеме представляет собой S-образную кривую:

Рисунок 1.41 S-образная кривая

Номиналы предохранителей

Предохранители

, как и следовало ожидать, в основном рассчитаны на ток: ампер. Хотя их работа зависит от самовыделения тепла в условиях чрезмерного тока за счет собственного электрического сопротивления предохранителя, они спроектированы так, чтобы вносить незначительное дополнительное сопротивление в цепи, которые они защищают.Это в значительной степени достигается за счет того, что плавкий провод делается как можно короче. Точно так же, как допустимая токовая нагрузка обычного провода не связана с его длиной (сплошной медный провод 10 калибра выдержит ток 40 ампер на открытом воздухе, независимо от длины или короткого отрезка), плавкий провод из определенного материала и калибра будет дуть при определенном токе независимо от того, как долго он длится. Поскольку длина не является фактором текущего рейтинга, чем короче она может быть сделана, тем меньшее сопротивление будет между концом и концом.

Однако разработчик предохранителя также должен учитывать, что происходит после сгорания предохранителя: оплавленные концы сплошного провода будут разделены воздушным зазором с полным напряжением питания между концами.Если предохранитель недостаточно длинный в цепи высокого напряжения, искра может перескочить с одного из концов расплавленного провода на другой, снова замкнув цепь:

Рисунок 1.42 Принципиальная схема конструктора предохранителей Рисунок 1.43 Принципиальная схема конструктора предохранителей

Следовательно, предохранители рассчитываются с точки зрения их допустимого напряжения, а также уровня тока, при котором они сработают.

Некоторые большие промышленные предохранители имеют заменяемые проволочные элементы для снижения затрат. Корпус предохранителя представляет собой непрозрачный картридж многоразового использования, защищающий провод предохранителя от воздействия и экранирующий окружающие предметы от провода предохранителя.

Номинальный ток предохранителя — это нечто большее, чем просто цифра. Если через предохранитель на 30 ампер пропускается ток в 35 ампер, он может внезапно перегореть или с задержкой перед перегоранием, в зависимости от других аспектов его конструкции. Некоторые предохранители предназначены для очень быстрого срабатывания, в то время как другие рассчитаны на более скромное время «открытия» или даже на замедленное срабатывание в зависимости от области применения. Последние иногда называют плавкими предохранителями с задержкой срабатывания из-за их преднамеренных характеристик задержки срабатывания.

Классическим примером применения плавкого предохранителя с задержкой срабатывания является защита электродвигателя, где пусковой ток обычно в десять раз превышает нормальный рабочий ток каждый раз, когда двигатель запускается с полной остановки. Если бы в таком приложении использовались быстродействующие предохранители, двигатель никогда бы не запустился, потому что при нормальных уровнях пускового тока предохранитель (и) немедленно перегорел бы! Конструкция плавкого предохранителя с задержкой срабатывания такова, что элемент плавкого предохранителя имеет большую массу (но не большую допустимую нагрузку), чем эквивалентный быстродействующий плавкий предохранитель, что означает, что он будет нагреваться медленнее (но до той же конечной температуры) при любом заданном количестве. тока.

На другом конце спектра действия предохранителей находятся так называемые полупроводниковые предохранители , предназначенные для очень быстрого размыкания в случае перегрузки по току. Полупроводниковые устройства, такие как транзисторы, как правило, особенно нетерпимы к условиям перегрузки по току и, как таковые, требуют быстродействующей защиты от сверхтоков в мощных приложениях.

Предохранители

всегда должны размещаться на «горячей» стороне нагрузки в заземленных системах. Это сделано для того, чтобы нагрузка была полностью обесточена во всех отношениях после срабатывания предохранителя.Чтобы увидеть разницу между плавлением «горячей» стороны и «нейтральной» стороны нагрузки, сравните эти две схемы:

Рисунок 1.44 Принципиальная схема конструктора предохранителей Рисунок 1.45 Принципиальная схема конструктора предохранителей

В любом случае предохранитель успешно прервал ток в нагрузке, но нижняя цепь не может прервать потенциально опасное напряжение с обеих сторон нагрузки на землю, где может стоять человек. . Первая схема намного безопаснее.

Как было сказано ранее, предохранители — не единственный используемый тип устройства защиты от сверхтоков.Устройства, похожие на выключатели, называемые автоматическими выключателями , часто (и чаще) используются для размыкания цепей с чрезмерным током, их популярность связана с тем, что они не разрушают себя в процессе размыкания цепи, как предохранители. В любом случае, однако, размещение устройства защиты от сверхтоков в цепи будет соответствовать тем же общим рекомендациям, перечисленным выше: а именно, «предохранить» сторону источника питания , а не , подключенную к земле.

Хотя размещение защиты от перегрузки по току в цепи может определять относительную опасность поражения электрическим током в этой цепи при различных условиях, следует понимать, что такие устройства никогда не предназначались для защиты от поражения электрическим током.Ни предохранители, ни автоматические выключатели не предназначены для срабатывания в случае поражения электрическим током; скорее, они предназначены для открытия только в условиях потенциального перегрева проводника. Устройства максимального тока в первую очередь защищают проводники цепи от повреждения при перегреве (и опасности возгорания, связанной с чрезмерно горячими проводниками) и, во вторую очередь, защищают определенные части оборудования, такие как нагрузки и генераторы (некоторые быстродействующие предохранители предназначены для защиты особенно чувствительных электронных устройств. к скачкам тока).Поскольку уровни тока, необходимые для поражения электрическим током или поражения электрическим током, намного ниже нормальных уровней тока обычных силовых нагрузок, состояние перегрузки по току не указывает на возникновение поражения электрическим током. Существуют и другие устройства, предназначенные для обнаружения определенных условий удара (детекторы замыкания на землю являются наиболее популярными), но эти устройства строго служат этой единственной цели и не связаны с защитой проводов от перегрева.

  • Плавкий предохранитель представляет собой небольшой тонкий проводник, предназначенный для плавления и разделения на две части с целью размыкания цепи в случае чрезмерного тока.
  • Автоматический выключатель — это специально разработанный переключатель, который автоматически размыкается для прерывания тока цепи в случае перегрузки по току. Они могут срабатывать (размыкаться) термически, магнитными полями или внешними устройствами, называемыми «реле защиты», в зависимости от конструкции выключателя, его размера и области применения.
  • Предохранители
  • в первую очередь рассчитаны на максимальный ток, но также рассчитаны на то, какое падение напряжения они будут безопасно выдерживать после прерывания цепи.
  • Предохранители
  • могут быть спроектированы так, чтобы срабатывать быстро, медленно или где-то посередине при одинаковом максимальном уровне тока.
  • Лучшее место для установки предохранителя в заземленной энергосистеме — на пути незаземленного проводника к нагрузке. Таким образом, при перегорании предохранителя к нагрузке останется только заземленный (безопасный) провод, что сделает безопаснее для людей находиться рядом.

Учебное пособие по физике: Заземление — снятие заряда

В предыдущих трех разделах Урока 2 обсуждались три распространенных метода зарядки — заряд трением, заряд индукцией и заряд проводимостью.Обсуждение зарядки было бы неполным без обсуждения uncharging . У объектов с избыточным зарядом — положительным или отрицательным — этот заряд может быть удален с помощью процесса, известного как заземление. Заземление — это процесс удаления избыточного заряда с объекта посредством передачи электронов между ним и другим объектом значительного размера. Когда заряженный объект заземлен, избыточный заряд уравновешивается переносом электронов между заряженным объектом и землей.Земля — это просто объект, который служит, казалось бы, бесконечным резервуаром электронов; Земля способна передавать электроны заряженному объекту или принимать электроны от заряженного объекта, чтобы нейтрализовать этот объект. В этом последнем разделе Урока 2 будет обсуждаться процесс заземления.

Заземление отрицательно заряженного объекта

Чтобы начать обсуждение заземления, мы рассмотрим заземление отрицательно заряженного электроскопа.Любой отрицательно заряженный объект имеет избыток электронов. Если нужно удалить заряд, ему придется потерять лишние электроны. Как только лишние электроны удалены из объекта, в объекте будет равное количество протонов и электронов, и он будет иметь баланс заряда. Чтобы удалить избыток электронов из отрицательно заряженного электроскопа, электроскоп должен быть подключен проводящим путем к другому объекту, который способен принимать эти электроны.Другой объект — земля. В типичных электростатических экспериментах и ​​демонстрациях это делается простым прикосновением к электроскопу рукой. При контакте избыточные электроны покидают электроскоп и попадают в человека, который его касается. Эти избыточные электроны впоследствии распространяются по поверхности человека.

Этот процесс заземления работает, потому что избыточные электроны отталкивают друг друга. Как всегда, отталкивающее воздействие между одноименно заряженными электронами заставляет их искать средства пространственного разделения друг от друга.Это пространственное разделение достигается за счет перемещения к более крупному объекту, который дает большую площадь поверхности для распространения. Из-за относительного размера человека по сравнению с типичным электроскопом избыточные электроны (почти все они) способны уменьшить силы отталкивания, перемещаясь в человека (то есть на землю). Как и контактная зарядка, о которой говорилось ранее, заземление — это просто еще один пример разделения заряда между двумя объектами. Степень, в которой объект готов разделить избыточный заряд, пропорциональна его размеру.Таким образом, эффективная земля — ​​это просто объект с достаточно значительным размером, чтобы разделить подавляющее большинство избыточного заряда.

Заземление положительно заряженного объекта

Предыдущее обсуждение описывает заземление отрицательно заряженного электроскопа. Электроны переносились с электроскопа на землю. Но что, если электроскоп заряжен положительно? Как перенос электрона позволяет нейтрализовать объект с избытком протонов? Чтобы изучить эти вопросы, мы рассмотрим заземление положительно заряженного электроскопа.Положительно заряженный электроскоп должен получать электроны, чтобы получить равное количество протонов и электронов. Собирая электроны от к земле , электроскоп будет иметь баланс заряда и, следовательно, будет нейтральным. Таким образом, заземление положительно заряженного электроскопа включает передачу электронов от земли к электроскопу. Этот процесс работает, потому что избыточный положительный заряд на электроскопе притягивает электроны от земли (в данном случае от человека).Хотя это может нарушить любой баланс заряда, присутствующий на человеке, значительно больший размер человека позволяет избыточному заряду отдаляться друг от друга. Как и в случае заземления отрицательно заряженного электроскопа, заземление положительно заряженного электроскопа включает разделение заряда. Избыточный положительный заряд распределяется между электроскопом и землей. И еще раз: степень, в которой объект готов разделить избыточный заряд, пропорциональна его размеру.Человек — эффективная почва, потому что у него достаточно размера, чтобы разделить подавляющее большинство избыточного положительного заряда.

Необходимость проведения пути

Любой объект может быть заземлен при условии, что заряженные атомы этого объекта имеют проводящий путь между атомами и землей. Обычно в лаборатории приклеивают две соломинки к заряженной алюминиевой пластине. Одна соломка покрыта алюминиевой фольгой, а другая — голым пластиком.При прикосновении к соломке с алюминиевым покрытием алюминиевая пластина теряет заряд. Он заземлен за счет движения электронов от земли к алюминиевой пластине. При прикосновении к пластиковой соломке заземления не происходит. Пластик служит изолятором и предотвращает попадание электронов от земли к алюминиевой пластине. Заземление требует наличия проводящего пути между землей и заземляемым объектом. Электроны будут двигаться по этому пути.

Урок 2 этого раздела Физического класса был посвящен методам зарядки и разрядки объектов.Один из принципов, который постоянно возникал, заключался в соотношении силы и расстояния. Эта связь будет исследована в Уроке 3.


Мы хотели бы предложить … Иногда просто прочитать об этом недостаточно. Вы должны с ним взаимодействовать! И это именно то, что вы делаете, когда используете один из интерактивных материалов The Physics Classroom. Мы хотели бы предложить вам совместить чтение этой страницы с использованием нашего интерактивного зарядного устройства.Вы можете найти его в разделе Physics Interactives на нашем сайте. Charging Interactive — это электростатическая «игровая площадка», которая позволяет учащемуся исследовать различные концепции, связанные с зарядом, взаимодействиями зарядов, процессами зарядки и заземлением. Как только вы освоитесь с концепциями, коснитесь кнопки «Играть» своим игровым лицом.

Проверьте свое понимание

Используйте свое понимание заряда, чтобы ответить на следующие вопросы.По завершении нажмите кнопку, чтобы просмотреть ответы.

1. Человек, стоящий на земле, касается положительно заряженной консервной банки. После этого поп может стать нейтральным. Поп может стать нейтральным во время этого процесса, потому что ______.

а. электроны переходят от баночки к человеку (земле)

г. электроны переходят от человека (земли) к банке

г. протоны переходят от баллончика к человеку (земле)

г.протоны переходят от человека (земли) к банке

2. Студент-физик, стоя на земле, касается разряженной пластиковой бейсбольной битой отрицательно заряженным электроскопом. Это вызовет ___.

а.

Добавить комментарий

Ваш адрес email не будет опубликован. Обязательные поля помечены *